Exam 2 Med3 Flashcards

1
Q

A patient with a spinal cord injury at the T1 level complains of a severe headache and an “anxious feeling.” Which is the most appropriate initial reaction by the nurse?

  1. Try to calm the patient and make the environment soothing.
  2. Assess for a full bladder.
  3. Notify the healthcare provider.
  4. Prepare the patient for diagnostic radiography.
A
  1. Assess for a full bladder.

Rationale: Autonomic dysreflexia occurs in patients with injury at level T6 or higher, and is a life-threatening situation that will require immediate intervention or the patient will die. The most common cause is an overextended bladder or bowel. Symptoms include hypertension, headache, diaphoresis, bradycardia, visual changes, anxiety, and nausea. A calm, soothing environment is fine, though not what the patient needs in this case. The nurse should recognize this as an emergency and proceed accordingly. Once the assessment has been completed, the findings will need to be communicated to the healthcare provider.

How well did you know this?
1
Not at all
2
3
4
5
Perfectly
2
Q

A hospitalized patient with a C7 cord injury begins to yell “I can’t feel my legs anymore.” Which is the most appropriate action by the nurse?

  1. Remind the patient of her injury and try to comfort her.
  2. Call the healthcare provider and get an order for radiologic evaluation.
  3. Prepare the patient for surgery, as her condition is worsening.
  4. Explain to the patient that this could be a common, temporary problem.
A
  1. Explain to the patient that this could be a common, temporary problem.

Rationale: Spinal shock is a condition almost half the people with acute spinal injury experience. It is characterized by a temporary loss of reflex function below level of injury, and includes the following symptomatology: flaccid paralysis of skeletal muscles, loss of sensation below the injury, and possibly bowel and bladder dysfunction and loss of ability to perspire below the injury level. In this case, the nurse should explain to the patient what is happening.

How well did you know this?
1
Not at all
2
3
4
5
Perfectly
3
Q

The nurse is caring for a patient with increased intracranial pressure (IICP). The nurse realizes that some nursing actions are contraindicated with IICP. Which nursing action should be avoided?

  1. Reposition the patient every two hours.
  2. Position the patient with the head elevated 30 degrees.
  3. Suction the airway every two hours per standing orders.
  4. Provide continuous oxygen as ordered.
A
  1. Suction the airway every two hours per standing orders.

Rationale: Suctioning further increases intracranial pressure; therefore, suctioning should be done to maintain a patent airway but not as a matter of routine. Maintaining patient comfort by frequent repositioning as well as keeping the head elevated 30 degrees will help to prevent (or even reduce) IICP. Keeping the patient properly oxygenated may also help to control ICP.

How well did you know this?
1
Not at all
2
3
4
5
Perfectly
4
Q

A patient with a spinal cord injury (SCI) is admitted to the unit and placed in traction. Which of the following actions is the nurse responsible for when caring for this patient?
Select all that apply.
1. modifying the traction weights as needed
2. assessing the patient’s skin integrity
3. applying the traction upon admission
4. administering pain medication
5. providing passive range of motion

A
  1. assessing the patient’s skin integrity
  2. administering pain medication
  3. providing passive range of motion

Rationale: The healthcare provider is responsible for initial applying of the traction device. The weights on the traction device must not be changed without the order of a healthcare provider. When caring for a patient in traction, the nurse is responsible for assessment and care of the skin due to the increased risk of skin breakdown. The patient in traction is likely to experience pain and the nurse is responsible for assessing this pain and administering the appropriate analgesic as ordered. Passive range of motion helps prevent contractures; this is often performed by a physical therapist or a nurse.

How well did you know this?
1
Not at all
2
3
4
5
Perfectly
5
Q
A patient has manifestations of autonomic dysreflexia. Which of these assessments would indicate a possible cause for this condition?
Select all that apply.
1. hypertension
2. kinked catheter tubing
3. respiratory wheezes and stridor
4. diarrhea
5. fecal impaction
A
  1. kinked catheter tubing
  2. fecal impaction

Rationale: Autonomic dysreflexia can be caused by kinked catheter tubing allowing the bladder to become full, triggering massive vasoconstriction below the injury site, producing the manifestations of this process. Acute symptoms of autonomic dysreflexia, including a sustained elevated blood pressure, may indicate fecal impaction. The other answers will not cause autonomic dysreflexia.

How well did you know this?
1
Not at all
2
3
4
5
Perfectly
6
Q

An unconscious patient receiving emergency care following an automobile crash accident has a possible spinal cord injury. What guidelines for emergency care will be followed?
Select all that apply.
1. Immobilize the neck using rolled towels or a cervical collar.
2. The patient will be placed in a supine position
3. The patient will be placed on a ventilator.
4. The head of the bed will be elevated.
5. The patient’s head will be secured with a belt or tape secured to the stretcher.

A
  1. Immobilize the neck using rolled towels or a cervical collar.
  2. The patient will be placed in a supine position
  3. The patient’s head will be secured with a belt or tape secured to the stretcher.

Rationale: In the emergency setting, all patients who have sustained a trauma to the head or spine, or are unconscious should be treated as though they have a spinal cord injury. Immobilizing the neck, maintaining a supine position and securing the patient’s head to prevent movement are all basic guidelines of emergency care. Placement on the ventilator and raising the head of the bed will be considered after admittance to the hospital.

How well did you know this?
1
Not at all
2
3
4
5
Perfectly
7
Q

A patient with a spinal cord injury is recovering from spinal shock. The nurse realizes that the patient should not develop a full bladder because what emergency condition can occur if it is not corrected quickly?

  1. autonomic dysreflexia
  2. autonomic crisis
  3. autonomic shutdown
  4. autonomic failure
A
  1. autonomic dysreflexia

Rationale: Be attuned to the prevention of a distended bladder when caring for spinal cord injury (SCI) patients in order to prevent this chain of events that lead to autonomic dysreflexia. Track urinary output carefully. Routine use of bladder scanning can help prevent the occurrence. Other causes of autonomic dysreflexia are impacted stool and skin pressure. Autonomic crisis, autonomic shutdown, and autonomic failure are not terms used to describe common complications of spinal injury associated with bladder distension.

How well did you know this?
1
Not at all
2
3
4
5
Perfectly
8
Q

Which patient is at highest risk for a spinal cord injury?

  1. 18-year-old male with a prior arrest for driving while intoxicated (DWI)
  2. 20-year-old female with a history of substance abuse
  3. 50-year-old female with osteoporosis
  4. 35-year-old male who coaches a soccer team
A
  1. 18-year-old male with a prior arrest for driving while intoxicated (DWI)

Rationale: The three major risk factors for spinal cord injuries (SCI) are age (young adults), gender (higher incidence in males), and alcohol or drug abuse. Females tend to engage in less risk-taking behavior than young men.

How well did you know this?
1
Not at all
2
3
4
5
Perfectly
9
Q

The nurse understands that when the spinal cord is injured, ischemia results and edema occurs. How should the nurse explain to the patient the reason that the extent of injury cannot be determined for several days to a week?

  1. “Tissue repair does not begin for 72 hours.”
  2. “The edema extends the level of injury for two cord segments above and below the affected level.”
  3. “Neurons need time to regenerate so stating the injury early is not predictive of how the patient progresses.”
  4. “Necrosis of gray and white matter does not occur until days after the injury.”
A
  1. “The edema extends the level of injury for two cord segments above and below the affected level.”

Rationale: Within 24 hours necrosis of both gray and white matter begins if ischemia has been prolonged and the function of nerves passing through the injured area is lost. Because the edema extends above and below the area affected, the extent of injury cannot be determined until after the edema is controlled. Neurons do not regenerate, and the edema is the factor that limits the ability to predict extent of injury.

How well did you know this?
1
Not at all
2
3
4
5
Perfectly
10
Q

A patient with a spinal cord injury (SCI) has complete paralysis of the upper extremities and complete paralysis of the lower part of the body. The nurse should use which medical term to adequately describe this in documentation?

  1. hemiplegia
  2. paresthesia
  3. paraplegia
  4. quadriplegia
A
  1. quadriplegia
    Rationale: Quadriplegia describes complete paralysis of the upper extremities and complete paralysis of the lower part of the body. Hemiplegia describes paralysis on one side of the body. Paresthesia does not indicate paralysis. Paraplegia is paralysis of the lower body.
How well did you know this?
1
Not at all
2
3
4
5
Perfectly
11
Q

Which of the following nursing actions is appropriate for preventing skin breakdown in a patient who has recently undergone a laminectomy?

  1. Provide the patient with an air mattress.
  2. Place pillows under patient to help patient turn.
  3. Teach the patient to grasp the side rail to turn.
  4. Use the log roll to turn the patient to the side.
A
  1. Use the log roll to turn the patient to the side.

Rationale: A patient who has undergone a laminectomy needs to be turned by log rolling to prevent pressure on the area of surgery. An air mattress will help prevent skin breakdown but the patient still needs to be turned frequently. Placing pillows under the patient can help take pressure off of one side but the patient still needs to change positions often. Teaching the patient to grasp the side rail will cause the spine to twist, which needs to be avoided.

How well did you know this?
1
Not at all
2
3
4
5
Perfectly
12
Q

The patient is admitted with injuries that were sustained in a fall. During the nurse’s first assessment upon admission, the findings are: blood pressure 90/60 (as compared to 136/66 in the emergency department), flaccid paralysis on the right, absent bowel sounds, zero urine output, and palpation of a distended bladder. These signs are consistent with which of the following?

  1. paralysis
  2. spinal shock
  3. high cervical injury
  4. temporary hypovolemia
A
  1. spinal shock

Rationale: Spinal shock is common in acute spinal cord injuries. In addition to the signs and symptoms mentioned, the additional sign of absence of the cremasteric reflex is associated with spinal shock. Lack of respiratory effort is generally associated with high cervical injury. The findings describe paralysis that would be associated with spinal shock in an spinal injured patient. The likely cause of these findings is not hypovolemia, but rather spinal shock.

How well did you know this?
1
Not at all
2
3
4
5
Perfectly
13
Q

While caring for the patient with spinal cord injury (SCI), the nurse elevates the head of the bed, removes compression stockings, and continues to assess vital signs every two to three minutes while searching for the cause in order to prevent loss of consciousness or death. By practicing these interventions, the nurse is avoiding the most dangerous complication of autonomic dysreflexia, which is which of the following?

  1. hypoxia
  2. bradycardia
  3. elevated blood pressure
  4. tachycardia
A
  1. elevated blood pressure

Rationale: Autonomic dysreflexia is an emergency that requires immediate assessment and intervention to prevent complications of extremely high blood pressure. Additional nursing assistance will be needed and a colleague needs to reach the physician stat.

How well did you know this?
1
Not at all
2
3
4
5
Perfectly
14
Q

A patient is admitted to the hospital with a CD4 spinal cord injury after a motorcycle collision. The patient’s BP is 83/49, and his pulse is 39 beats/min, and he remains orally intubated. The nurse identifies this pathophysiologic response as caused by

a. increased vasomotor tone after injury
b. a temporary loss of sensation and flaccid paralysis below the level of injury
c. loss of parasympathetic nervous system innervation resulting in vasoconstriction
d. loss of sympathetic nervous system innervation resulting in peripheral vasodilation

A

D. loss of sympathetic nervous system innervation resulting in peripheral vasodilation

How well did you know this?
1
Not at all
2
3
4
5
Perfectly
15
Q

A nurse is caring for a client with a spinal cord injury who reports a severe headache and is sweating profusely. vital signs include BP 220/110, apical heart rate of 54/min. Which of the following acctions should the nurse take first?

a. notify the provider
b. sit the client upright in bed
c. check the client’s urinary catheter for blockage
d. administer antihypertensive medication

A

B. sit the client upright in bed

Rationale: The greatest risk to the client is experiencing a cerebrovascular accident (stroke) secondary to elevated BP. The first action by the nurse is elevate the head of the bed until the client is in an upright position. this will lower the BP secondary to postural hypotension.

How well did you know this?
1
Not at all
2
3
4
5
Perfectly
16
Q

Following a T2 spinal cord injury, the patient develops paralytic ileus. While this condition is present, the nurse anticipates that the patient will need

a. IV fluids
b. tube feedings
c. parenteral nutrition
d. nasogastric suctioning

A

D. nasogastric suctioning

Rationale: During the first 2 to 3 days after a spinal cord injury, paralytic ileus may occur, and NG suction must be used to remove secretions and gas from the GI tract until peristalsis resumes. IV fluids are used to maintain fluid balance but do not specifically relate to paralytic ileus. Tube feedings would be used only for patients who had difficulty swallowing and not until peristalsis is returned; PN would be used only if the paralytic ileus was unusally prolonged.

How well did you know this?
1
Not at all
2
3
4
5
Perfectly
17
Q

An initial incomplete spinal cord injury often results in complete cord damage because of

a. edematous compression of the cord above the level of the injury
b. continued trauma to the cord resulting from damage to stabilizing ligaments
c. infarction and necrosis of the cord caused by edema, hemorrhage, and metabolites
d. mecheanical transection of the cord by sharp vertebral bone fragments after the initial injury

A

C. infarction and necrosis of the cord caused by edema, hemorrhage, and metabolites

Rationale: The primary injury of the spinal cord rarely affects the entire cord, but the patho of secondary injury may result in damage that is the same as mechanical severance of the cord. Complete cord dissolution occurs through autodestruction of the cord by hemorrhage, edema, and the presence of metabolites and norepinephrine. resulting in anoxia and infarction of the cord. Edema resulting from the inflammatory response may increase the damage as it extends above and below the injury site.

How well did you know this?
1
Not at all
2
3
4
5
Perfectly
18
Q

Two days following a spinal cord injury, a patient asks continually about the extent of impairment that will result from the injury. The best response by the nurse is,

a. you will have more normal function when spinal shock resolves and the reflex arc returns
b. the extent of your injury cannot be determined until the secondary injury to the cord is resolved
c. when your condition is more stable, an MRI will be done that can reveal the extent of the cord damage
d. because long-term rehabilitation can affect the return of tunction, it will be years before we can tell when the complete effect will be

A

B. the extent of your injury cannot be determined until the secondary injury to the cord is resolved

Rationale: Until the edema and necrosis at the site of the injury are resolved in 72 hours to 1 week after the injury, it is not possible to determine how much cord damage is present from the initial injury, how much secondary injury occurred, or how much the cord was damaged by edema that extended above the level of the original injury. The return of reflexes signals only the end of spinal shock, and the reflexes may be inappropriate and excessive, causing spasms that complicate rehab.

How well did you know this?
1
Not at all
2
3
4
5
Perfectly
19
Q

A week following a spinal cord injury at T2, a patient experiences movement in his leg and tells the nurse he is recovering some function. The nurses’ best response to the patient is,

a. it is really still too soon to know if you will have a return of function
b. the could be a really positive finding. can you show me the movement
c. that’s wonderful. we will start exercising your legs more frequently now
d. im sorry, but the movement is only a reflex and does not indicate normal function

A

B. the could be a really positive finding. can you show me the movement

Rationale: in 1 week following a spinal cord injury, there may be a resolution of the edema of the injury and an end to spinal shock. When spinal shock ends, reflex movement and spasms will occur, which may be mistaken for return of function, but with the resolution of edema, some normal function may also occur. it is important when movement occurs to determine whether the movement is voluntary and can be consciously controlled, which would indicate some return of function.

How well did you know this?
1
Not at all
2
3
4
5
Perfectly
20
Q

Urinary function during the acute phase of spinal cord injury is maintained with

a. an indwelling catheter
b. intermittent catheterization
c. insertion of a suprapubic catheter
d. use of incontinent pads to protect the skin

A

A. an indwelling catheterization

How well did you know this?
1
Not at all
2
3
4
5
Perfectly
21
Q

A nurse is caring for a client who has a C4 spinal cord injury. which of the following should the nurse recognize the client as being at the greatest risk for?

a. neurogenic shock
b. paralytic ileus
c. stress ulcer
d. respiratory compromise

A

D. respiratory compromise

Rationale: Using the airway, breathing and circulation priority framework, the greatest risk to the client with a SCI at the level of C4 is respiratory compromise secondary to involvement of the phrenic nerve. Maintenance of an airway and provision of ventilator support as needed is the priority intervention.

How well did you know this?
1
Not at all
2
3
4
5
Perfectly
22
Q

During assessment of a patient with a spinal cord injury, the nurse determines that the patient has a poor cough with diaphragmatic breathing. Based on this finding, the nurses’ first action should be to

a. initiate frequent turning and repositioning
b. use tracheal suctioning to remove secretions
c. assess lung sounds and respiratory rate and depth
d. prepare the patient for endotracheal intubation and mechanical ventilation

A

C. assess lungs sounds and respiratory rate and depth

Rationale: Because pneumonia and atelectasis are potential problems RT ineffective coughing function, the nurse should assess the patient’s breath sound and resp function to determine whether secretions are being retained or whether there is progression of resp impairment. Suctioning is not indicated unless lung sounds indicate retained secretions: position changes will help mobilize secretions. Intubation and mechanical ventilation are used if the patient becomes exhausted from labored breathing or if ABGs deteriorate.

How well did you know this?
1
Not at all
2
3
4
5
Perfectly
23
Q

The healthcare provider has ordered IV dopamine (Intropin) for a patient in the emergency deparement with a spinal cord injury. The nurse determines that the drug is having the desired effect when assessment findings include

a. pulse rate of 68
b. respiratory rate of 24
c. BP of 106/82
d. temperature of 96.8

A

C. BP of 106/82
Rationale: Dopamine is a vasopressor that is used to maintain BP during states of hypotension that occur during neurogenic shock associated with spinal cord injury. Atropine would be used to treat bradycardia. The T reflects some degree of poikilothermism, but this is not treated with medications.

How well did you know this?
1
Not at all
2
3
4
5
Perfectly
24
Q

A patient is admitted to the emergency department with a spinal cord injury at the level of T2. Which of the following findings is of most concern to the nurse?

a. SpO2 of 92%
b. HR of 42 beats/min
c. BP of 88/60
d. loss of motor and sensory function in arms and legs

A

b. HR of 42 beats/min
Rationale: Neurogenic shock associated with cord injuries above the level of T6 greatly decrease the effect of the sympathetic nervous system, and bradycardia and hypotension occur. A heart rate of 42 is not adequate to meet oxygen needs of the body, and while low, the BP is not at a critical point. The O2 sat is ok, and the motor and sensory loss are expected.

How well did you know this?
1
Not at all
2
3
4
5
Perfectly
25
Q

Without surgical stabilization, immobilization and traction of the patient with a cervical spinal cord injury most frequently requires the use of

a. kinetic beds
b. hard cervical collars
c. skeletal traction with skull tongs
d. sternal-occipital-mandibular immobilizer (SOMI) brace

A

C. skeletal traction with skull tongs
Rationale: Cervical injuries usually require skeletal traction with the use of Crutchfield, Vinke, or other types of skull tongs to immobilize the cervical vertebrae, even if fracture has not occurred. Hard cervical collars are used for minor injuries or for stabilization during emergency transport of the patient. Sandbags are also used temporarily to stabilize the neck during insertion of tongs or during diagnostic testing immediately following the injury. Special turning or kinetic beds may be used to turn and mobilize patients who are in cervical traction.

How well did you know this?
1
Not at all
2
3
4
5
Perfectly
26
Q

A patient with a spinal cord injury has spinal shock. The nurse plans care for the patient based on the knowledge that

a. rehabilitation measures cannot be initiated until spinal shock has resolved
b. the patient will need continuous monitoring for hypotension, tachycardia, and hypoxemia
c. resolution of spinal shock is manifested by spasticity, hyperreflexia, and reflex emptying of the bladder
d. the patient will have complete loss of motor and sensory functions below the level of the injury, but autonomic functions are not affected

A

C. c. resolution of spinal shock is manifested by spasticity, hyperreflexia, and reflex emptying of the bladder
Rationale: Spinal shock occurs in about half of all people with acute spinal cord injury. In spinal shock, the entire cord below the level of the lesion fails to function, resulting in a flaccid paralysis and hypomotility of most processes without any reflex activity. Return of reflex activity signals the end of spinal shock. Sympathetic function is impaired belwo the level of the injury because sympathetic nerves leave the spinal cord at the thoracic and lumbar areas, and cranial parasympathetic nerves predominate in control over respirations, heart, and all vessels and organ below the injury. Neurogenic shock results from loss of vascular tone caused by the injury and is manifested by hypotension, peripheral vasodilation, and decreased CO. Rehab activities are not contraindicated during spainl shock and should be instituted if the patient’s cardiopulmonary status is stable.

How well did you know this?
1
Not at all
2
3
4
5
Perfectly
27
Q

A patient with a C7 spinal cord injury undergoing rehabilitation tells the nurse he must have the flu because he has a bad headache and nausea. The initial action of the nurse is to

a. call the physician
b. check the patient’s temperature
c. take the patient’s BP
d. elevate the HOB to 90 degrees

A

c. Take the patient’s BP

How well did you know this?
1
Not at all
2
3
4
5
Perfectly
28
Q

One indication for surgical therapy of the patient with a spinal cord injury is when

a. there is incomplete cord lesion involvement
b. the ligaments that support the spine are torn
c. a high cervical injury causes loss of respiratory function
d. evidence of continued compression of the cord is apparent

A

D. evidence of continued compression of the cord is apparent
Rationale: Although surgical treatment of spinal cord injuries often depends on the preference of the health care provider, surgery is usually indicated when there is continued compression of the cord by extrinsic forces or when there is evidence of cord compression. Other indications may include progressive neurologic deficit, compound fracture of the vertebra, bony fragments, and penetrating wounds of the cord.

How well did you know this?
1
Not at all
2
3
4
5
Perfectly
29
Q

A patient is admitted to the emergency department with a possible cervical spinal cord injury following an automobile crash. During the admission of the patient, the nurse places the highest priority on

a. maintaining a patent airway
b. assessing the patient for head and other injuries
c. maintaining immobilization of the cervical spine
d. assessing the patient’s motor and sensory function

A

a. maintaining a patent airway
Rationale: The need for a patent airway is the first priority for any injured patient, and a high cervical injury may decrease the gag reflex and ability to maintain an airway, as well as the ability to breathe. Maintaining cervical stability is then a consideration, along with assessing for other injuries and the patients neuro status.

How well did you know this?
1
Not at all
2
3
4
5
Perfectly
30
Q

A nurse is planning care for a client who suffered a spinal cord injury (SCI) involving a T12 fracture 1 week ago. The client has no muscle control of the lower limbs, bowel, or bladder. which of the following should be the nurses’ greatest priority?

a. prevention of further damage to the spinal cord
b. prevention of contractures of the lower extremities
c. prevention of skin breakdown of areas that lack sensation
d. prevention of postural hypotension when placing the client in a wheelchair

A

A. prevention of further damage to the spinal cord

Rationale: The greatest risk to the client during the acute phase of a SCI is further damage to the spinal cord. Therefore, when planning care, the priority should be the prevention of further damage to the spinal cord by administration of corticosteroids, minimizing movement of the client until spinal stabilization is accomplished through either traction or surgery, and adequate oxygenation of the client to decrease ischemia of the spinal cord.

How well did you know this?
1
Not at all
2
3
4
5
Perfectly
31
Q

Goals of rehabilitation for the patient with an injury at the C6 level include (select all that apply)

a. stand erect with leg brace
b. feed self with hand devices
c. drive an electric wheelchair
d. assist with transfer activities
e. drive adapted van from wheelchair

A

b. feed self with hand devices
c. drive an electric wheelchair
d. assist with transfer activities
e. drive adapted van from wheelchair

How well did you know this?
1
Not at all
2
3
4
5
Perfectly
32
Q

A nurse is caring for a client who experienced a cervical spine injury 3 months ago. Which of the following types of bladder management methods should the nurse use for this client?

a. condom catheter
b. intermittent urinary catheterization
c. crede’s method
d. indwelling urinary catheter

A

a. condom catheter
Rationale: a client who has a cervical spinal cord injury will also have a upper motor neuron injury, which is manifested by a spastic bladder. because the bladder will empty on its own, a condom catheter is an appropriate method and is noninvasive.
B & C are for flaccid bladder.

How well did you know this?
1
Not at all
2
3
4
5
Perfectly
33
Q

A patient is admitted with a spinal cord injury at the C7 level. During assessment the nurse identifies the presence of spinal shock on finding

a. paraplegia with flaccid paralysis
b. tetraplegia with total sensory loss
c. total hemiplegia with sensory and motor loss
d. spastic tetraplegia with loss of pressure sensation

A

B. tetraplegia with total sensory loss
Rationale: At the C7 level, spinal shock is manifested by tetraplegia and sensory loss. The neurologic loss may be temporary or permanent. Paraplegia with sensory loss would occur at the level of T1. A hemiplegia occurs with central (brain) lesions affecting motor neurons and spastic tetraplegia occurs when spinal shock resolves.

How well did you know this?
1
Not at all
2
3
4
5
Perfectly
34
Q

During the patient’s process of grieving for the losses resulting from spinal cord injury, the nurse

a. helps the patient understand that working through the grief will be a lifelong process
b. should assist the patient to move through all stages of the mourning process to acceptance
c. lets the patient know that anger directed at the staff or the family is not a positive coping mechanism
d. facilitates the grieving process so that it is completed by the time the patient is discharged from rehabilitation

A

A. helps the patient understand that working through the grief will be a lifelong process

How well did you know this?
1
Not at all
2
3
4
5
Perfectly
35
Q

In planning community education for prevention of spinal cord injuries, the nurse targets

a. elderly men
b. teenage girls
c. elementary school-age children
d. adolescent and young adult men

A

D. adolescent and young adult men
Rationale: Spinnal cord injuries are highest in young adult men between the ages of 15 and 30 and those who are impulsive or risk takers in daily living. Other risk factors include alcohol and drug abuse as well as participation in sports and occupational exposure to trauma or violence.

How well did you know this?
1
Not at all
2
3
4
5
Perfectly
36
Q

In counseling patient with spinal cord lesions regarding sexual function, the nurse advises a male patient with a complete lower motor neuron lesion that he

a. is most likely to have reflexogenic erections and may experience orgasm if ejaculation occurs
b. may have uncontrolled reflex erections, but that orgasm and ejaculation are usually not possible
c. has a lesion with the greatest possibility of successful psychogenic erection with ejaculation and orgasm
d. will probably be unable to have either psychogenic or reflexogenic erections with no ejaculation or orgasm

A

D. will probably be unable to have either psychogenic or reflexogenic erections with no ejaculation or orgasm
Rationale: Most patients with a complete lower motor neuron lesion are unable to have either psychogenic or reflexogenic erections, and alterative methods of obtaining sexual satisfaction may be suggested. Patients with incomplete lower motor neuron lesions have the highest possibility of successful psychogenic erections with ejaculation, whereas patients with incomplete upper motor neuron lesions are more likey to experience reflexogeic erections with ejaculation. Patients with complete upper motor neuron lesions usually only have reflex sexual function with rare ejaculation.

How well did you know this?
1
Not at all
2
3
4
5
Perfectly
37
Q

A patient with paraplegia has developed an irritable bladder with reflex emptying. The nurse teaches the patient

a. hygiene care for an indwelling urinary catheter
b. how to perform intermittent self-catheterization
c. to empty the bladder with manual pelvic pressure in coordination with reflex voiding patterns
d. that a urinary diversion, such as an ileal conduit, is the easiest way to handle urinary elimination

A

B. b. how to perform intermittent self-catheterization
Rationale: Intermittent self cath five to six times a day is the recommended method of bladder management for the patient with a spinal cord injury because it more closely mimics normal emptying and has less potential for infectinon. The patient and family should be taught the procedure using clean technique, and if the patient has use of the arms, self-cath is use during the acute phase to prevent overdistention of the bladder and surgical urinary diversions are used if urinary complications occur.

How well did you know this?
1
Not at all
2
3
4
5
Perfectly
38
Q

A nurse is caring for a client who experienced a cervical spine injury 24 hours ago. which of the following types of prescribed medications should the nurse clarify with the provider?

a. glucocorticoids
b. plasma expanders
c. H2 antagonists
d. muscle relaxants

A

D. muscle relaxants

Rationale: The client will still be in spinal shock 24 hours following the injury. the client will not experience muscle spasms until after the spinal shock has resolved, making muscle relaxants unnecessary at this time.

How well did you know this?
1
Not at all
2
3
4
5
Perfectly
39
Q

When caring for a patient who was admitted 24 hours previously with a C5 spinal cord injury, which nursing action has the highest priority?

a. Continuous cardiac monitoring for bradycardia
b. Administration of methylprednisolone (Solu-Medrol) infusion
c. Assessment of respiratory rate and depth
d. Application of pneumatic compression devices to both legs

A

c. Assessment of respiratory rate and depth

Rationale: Edema around the area of injury may lead to damage above the C4 level, so the highest priority is assessment of the patient’s respiratory function. The other actions are also appropriate but are not as important as assessment of respiratory effort.

Cognitive Level: Application Text Reference: p. 1602
Nursing Process: Assessment NCLEX: Physiological Integrity

How well did you know this?
1
Not at all
2
3
4
5
Perfectly
40
Q

A 26-year-old patient with a C8 spinal cord injury tells the nurse, “My wife and I have always had a very active sex life, and I am worried that she may leave me if I cannot function sexually.” The most appropriate response by the nurse to the patient’s comment is to

a. advise the patient to talk to his wife to determine how she feels about his sexual function.
b. tell the patient that sildenafil (Viagra) helps to decrease erectile dysfunction in patients with spinal cord injury.
c. inform the patient that most patients with upper motor neuron injuries have reflex erections.
d. suggest that the patient and his wife work with a nurse specially trained in sexual counseling.

A

d. suggest that the patient and his wife work with a nurse specially trained in sexual counseling.

Rationale: Maintenance of sexuality is an important aspect of rehabilitation after spinal cord injury and should be handled by someone with expertise in sexual counseling. Although the patient should discuss these issues with his wife, open communication about this issue may be difficult without the assistance of a counselor. Sildenafil does assist with erectile dysfunction after spinal cord injury, but the patient’s sexuality is not determined solely by the ability to have an erection. Reflex erections are common after upper motor neuron injury, but these erections are uncontrolled and cannot be maintained during coitus.

Cognitive Level: Application Text Reference: p. 1608
Nursing Process: Implementation NCLEX: Psychosocial Integrity

How well did you know this?
1
Not at all
2
3
4
5
Perfectly
41
Q

A patient with a neck fracture at the C5 level is admitted to the intensive care unit (ICU) following initial treatment in the emergency room. During initial assessment of the patient, the nurse recognizes the presence of spinal shock on finding

a. hypotension, bradycardia, and warm extremities.
b. involuntary, spastic movements of the arms and legs.
c. the presence of hyperactive reflex activity below the level of the injury.
d. flaccid paralysis and lack of sensation below the level of the injury.

A

d. flaccid paralysis and lack of sensation below the level of the injury.

Rationale: Clinical manifestations of spinal shock include decreased reflexes, loss of sensation, and flaccid paralysis below the area of injury. Hypotension, bradycardia, and warm extremities are evidence of neurogenic shock. Involuntary spastic movements and hyperactive reflexes are not seen in the patient at this stage of spinal cord injury.

How well did you know this?
1
Not at all
2
3
4
5
Perfectly
42
Q

When caring for a patient who experienced a T1 spinal cord transsection 2 days ago, which collaborative and nursing actions will the nurse include in the plan of care? (Select all that apply.)

a. Endotracheal suctioning
b. Continuous cardiac monitoring
c. Avoidance of cool room temperature
d. Nasogastric tube feeding
e. Retention catheter care
f. Administration of H2 receptor blockers

A

b. Continuous cardiac monitoring
c. Avoidance of cool room temperature
d. Nasogastric tube feeding
e. Retention catheter care
f. Administration of H2 receptor blockers

Rationale: The patient is at risk for bradycardia and poikilothermia caused by sympathetic nervous system dysfunction and should have continuous cardiac monitoring and maintenance of a relatively warm room temperature. Gastrointestinal (GI) motility is decreased initially and NG suctioning is indicated. To avoid bladder distension, a retention catheter is used during this acute phase. Stress ulcers are a common complication but can be avoided through the use of the H2 receptor blockers such as famotidine.

How well did you know this?
1
Not at all
2
3
4
5
Perfectly
43
Q

A patient with a T1 spinal cord injury is admitted to the intensive care unit (ICU). The nurse will teach the patient and family that

a. use of the shoulders will be preserved.
b. full function of the patient’s arms will be retained.
c. total loss of respiratory function may occur temporarily.
d. elevations in heart rate are common with this type of injury.

A

b. full function of the patient’s arms will be retained.

Rationale: The patient with a T1 injury can expect to retain full motor and sensory function of the arms. Use of only the shoulders is associated with cervical spine injury. Total loss of respiratory function occurs with injuries above the C4 level and is permanent. Bradycardia is associated with injuries above the T6 level.

How well did you know this?
1
Not at all
2
3
4
5
Perfectly
44
Q

In which order will the nurse perform the following actions when caring for a patient with possible cervical spinal cord trauma who is admitted to the emergency department?

a. Administer O2 using a non-rebreathing mask.
b. Monitor cardiac rhythm and blood pressure.
c. Immobilize the patient’s head, neck, and spine.
d. Transfer the patient to radiology for spinal CT.

A

c. Immobilize the patient’s head, neck, and spine.
a. Administer O2 using a non-rebreathing mask.
b. Monitor cardiac rhythm and blood pressure.
d. Transfer the patient to radiology for spinal CT.

How well did you know this?
1
Not at all
2
3
4
5
Perfectly
45
Q

When caring for a patient who had a C8 spinal cord injury 10 days ago and has a weak cough effort, bibasilar crackles, and decreased breath sounds, the initial intervention by the nurse should be to

a. administer oxygen at 7 to 9 L/min with a face mask.
b. place the hands on the epigastric area and push upward when the patient coughs.
c. encourage the patient to use an incentive spirometer every 2 hours during the day.
d. suction the patient’s oral and pharyngeal airway

A

b. place the hands on the epigastric area and push upward when the patient coughs.

Rationale: The nurse has identified that the cough effort is poor, so the initial action should be to use assisted coughing techniques to improve the ability to mobilize secretions. Administration of oxygen will improve oxygenation, but the data do not indicate hypoxemia, and oxygen will not help expel respiratory secretions. The use of the spirometer may improve respiratory status, but the patient’s ability to take deep breaths is limited by the loss of intercostal muscle function. Suctioning may be needed if the patient is unable to expel secretions by coughing but should not be the nurse’s first action.

How well did you know this?
1
Not at all
2
3
4
5
Perfectly
46
Q

The nurse discusses long-range goals with a patient with a C6 spinal cord injury. An appropriate patient outcome is

a. transfers independently to a wheelchair.
b. drives a car with powered hand controls.
c. turns and repositions self independently when in bed.
d. pushes a manual wheelchair on flat, smooth surfaces.

A

d. pushes a manual wheelchair on flat, smooth surfaces.

Rationale: The patient with a C6 injury will be able to use the hands to push a wheelchair on flat, smooth surfaces. Because flexion of the thumb and fingers is minimal, the patient will not be able to grasp a wheelchair during transfer, drive a car with powered hand controls, or turn independently in bed.

How well did you know this?
1
Not at all
2
3
4
5
Perfectly
47
Q

A patient with a paraplegia resulting from a T10 spinal cord injury has a neurogenic reflex bladder. When the nurse develops a plan of care for this problem, which nursing action will be most appropriate?

a. Teaching the patient how to self-catheterize
b. Assisting the patient to the toilet q2-3hr
c. Use of the Credé method to empty the bladder
d. Catheterization for residual urine after voiding

A

a. Teaching the patient how to self-catheterize

Rationale: Because the patient’s bladder is spastic and will empty in response to overstretching of the bladder wall, the most appropriate method is to avoid incontinence by emptying the bladder at regular intervals through intermittent catheterization. Assisting the patient to the toilet will not be helpful because the bladder will not empty. The Credé method is more appropriate for a bladder that is flaccid, such as occurs with a reflexic neurogenic bladder. Catheterization after voiding will not resolve the patient’s incontinence.

How well did you know this?
1
Not at all
2
3
4
5
Perfectly
48
Q

Which is most important to respond to in a patient presenting with a T3 spinal injury?
A. Blood pressure of 88/60 mm Hg, pulse of 56 beats/minute
B. Deep tendon reflexes of 1+, muscle strength of 1+
C. Pain rated at 9
D. Warm, dry skin

A

A. Blood pressure of 88/60 mm Hg, pulse of 56 beats/minute
Neurogenic shock is a loss of vasomotor tone caused by injury, and it is characterized by hypotension and bradycardia. The loss of sympathetic nervous system innervations causes peripheral vasodilation, venous pooling, and a decreased cardiac output. The other options can be expected findings and are not as significant. Patients in neurogenic shock have pink and dry skin, instead of cold and clammy, but this sign is not as important as the vital signs.

How well did you know this?
1
Not at all
2
3
4
5
Perfectly
49
Q

The patient arrives in the emergency department from a motor vehicle accident, during which the car ran into a tree. The patient was not wearing a seat belt, and the windshield is shattered. What action is most important for you to do?
A. Determine if the patient lost consciousness.
B. Assess the Glasgow Coma Scale (GCS) score.
C. Obtain a set of vital signs.
D. Use a logroll technique when moving the patient.

A

D. Use a logroll technique when moving the patient.

When the head hits the windshield with enough force to shatter it, you must assume neck or cervical spine trauma occurred and you need to maintain spinal precautions. This includes moving the patient in alignment as a unit or using a logroll technique during transfers. The other options are important and are done after spinal precautions are applied.

How well did you know this?
1
Not at all
2
3
4
5
Perfectly
50
Q

One month after a spinal cord injury, which finding is most important for you to monitor?
A. Bladder scan indicates 100 mL.
B. The left calf is 5 cm larger than the right calf.
C. The heel has a reddened, nonblanchable area.
D. Reflux bowel emptying.

A

B. The left calf is 5 cm larger than the right calf.
Deep vein thrombosis is a common problem accompanying spinal cord injury during the first 3 months. Pulmonary embolism is one of the leading causes of death. Common signs and symptoms are absent. Assessment includes Doppler examination and measurement of leg girth. The other options are not as urgent to deal with as potential deep vein thrombosis.

How well did you know this?
1
Not at all
2
3
4
5
Perfectly
51
Q
Which clinical manifestation do you interpret as representing neurogenic shock in a patient with acute spinal cord injury?
A. Bradycardia
B. Hypertension
C. Neurogenic spasticity
D. Bounding pedal pulses
A

A. Bradycardia
Neurogenic shock results from loss of vasomotor tone caused by injury and is characterized by hypotension and bradycardia. Loss of sympathetic innervation causes peripheral vasodilation, venous pooling, and a decreased cardiac output.

How well did you know this?
1
Not at all
2
3
4
5
Perfectly
52
Q

A male client with a spinal cord injury is prone to experiencing automatic dysreflexia. The nurse would avoid which of the following measures to minimize the risk of recurrence?

a. Strict adherence to a bowel retraining program
b. Keeping the linen wrinkle-free under the client
c. Preventing unnecessary pressure on the lower limbs
d. Limiting bladder catheterization to once every 12 hours

A

d. Limiting bladder catheterization to once every 12 hours

The most frequent cause of autonomic dysreflexia is a distended bladder. Straight catheterization should be done every 4 to 6 hours, and foley catheters should be checked frequently to prevent kinks in the tubing. Constipation and fecal impaction are other causes, so maintaining bowel regularity is important. Other causes include stimulation of the skin from tactile, thermal, or painful stimuli. The nurse administers care to minimize risk in these areas.

How well did you know this?
1
Not at all
2
3
4
5
Perfectly
53
Q

A client with a spinal cord injury is prone to experiencing autonomic dysreflexia. The nurse should avoid which measure to minimize the risk of recurrence?

  1. strict adherence to a bowel retraining program
  2. keeping the linen wrinkle free under the client
  3. avoiding unnecessary pressure on the lower limbs
  4. limiting bladder catheterization to once every 12 hours
A
  1. limiting bladder cath to once q12h

(the most frequent cause of autonomic dysreflexia is a distended bladder . Straight cath should be performed q4-6 hrs and foley cath should be checked frequently for kinks in tubing . Constipation and fecal impaction are other causes, so maintaining bowel irregularity is important .

How well did you know this?
1
Not at all
2
3
4
5
Perfectly
54
Q

Which type of headache is suspected when the headaches are unilateral and throbbing, preceded by a prodrome of photophobia, and associated with a family history of this type of headache?

a. Cluster
b. Migraine
c. Frontal-type
d. Tension-type

A

b. Migraine

Migraine headaches are frequently unilateral and usually throbbing. They may be preceded by a prodrome and frequently there is a family history. Cluster headaches are also unilateral with severe bone-crushing pain but there is no prodrome or family history. Frontal-type headache is not a functional type of headache. Tension-type headaches are bilateral with constant, squeezing tightness without prodrome or family history.

How well did you know this?
1
Not at all
2
3
4
5
Perfectly
55
Q

A patient is diagnosed with cluster headaches. The nurse knows that which characteristics are associated with this type of headache (select all that apply)?

a. Family history
b. Alcohol is the only dietary trigger
c. Abrupt onset lasting 5 to 180 minutes
d. Severe, sharp, penetrating head pain
e. Bilateral pressure or tightness sensation
f. May be accompanied by unilateral ptosis or lacrimation

A

b. Alcohol is the only dietary trigger
c. Abrupt onset lasting 5 to 180 minutes
d. Severe, sharp, penetrating head pain
f. May be accompanied by unilateral ptosis or lacrimation

Cluster headaches have only alcohol as a dietary trigger and have an abrupt onset lasting 5 minutes to 3 hours with severe, sharp, penetrating pain. Cluster headaches may be accompanied by unilateral ptosis, lacrimation, rhinitis, facial flushing or pallor and commonly recur several times each day for several weeks, with months or years between clustered attacks. Family history and nausea, vomiting, or irritability may be seen with migraine headaches. Bilateral pressure occurring between migraine headaches and intermittent occurrence over long periods of time are characteristics of tension-type headaches.

How well did you know this?
1
Not at all
2
3
4
5
Perfectly
56
Q

What is the most important method of diagnosing functional headaches?

a. CT scan
b. Electromyography (EMG)
c. Cerebral blood flow studies
d. Thorough history of the headache

A

d. Thorough history of the headache

The primary way to diagnose and differentiate between headaches is with a careful history of the headaches, requiring assessment of specific details related to the headache. Electromyelography (EMG) may reveal contraction of the neck, scalp, or facial muscles in tensiontype headaches but this is not seen in all patients. CT scans and cerebral angiography are used to rule out organic causes of the headaches.

How well did you know this?
1
Not at all
2
3
4
5
Perfectly
57
Q

What drug therapy is included for acute migraine and cluster headaches that appears to alter the pathophysiologic process for these headaches?

a. β-Adrenergic blockers such as propranolol (Inderal)
b. Serotonin antagonists such as methysergide (Sansert)
c. Tricyclic antidepressants such as amitriptyline (Elavil)
d. Specific serotonin receptor agonists such as sumatriptan (Imitrex)

A

d. Specific serotonin receptor agonists such as sumatriptan (Imitrex)

Triptans (sumatriptan [Imitrex]) affect selected serotonin receptors that decrease neurogenic inflammation of the cerebral blood vessels and produce vasoconstriction. Both migraine headaches and cluster headaches appear to be related to vasodilation of cranial vessels and drugs that cause vasoconstriction are useful in treatment of migraine and cluster headaches. Methysergide blocks serotonin receptors in the central and peripheral nervous systems and is used for prevention of migraine and cluster headaches. β adrenergic blockers and tricyclic antidepressants are used prophylactically for migraine headaches but are not effective for cluster headaches.

How well did you know this?
1
Not at all
2
3
4
5
Perfectly
58
Q

What is a nursing intervention that is appropriate for the patient with a nursing diagnosis of anxiety related to lack of knowledge of the etiology and treatment of headache?

a. Help the patient to examine lifestyle patterns and precipitating factors.
b. Administer medications as ordered to relieve pain and promote relaxation.
c. Provide a quiet, dimly lit environment to reduce stimuli that increase muscle tension and anxiety.
d. Support the patient’s use of counseling or psychotherapy to enhance conflict resolution and stress reduction.

A

a. Help the patient to examine lifestyle patterns and precipitating factors.

When the anxiety is related to a lack of knowledge about the etiology and treatment of a headache, helping the patient to identify stressful lifestyle patterns and other precipitating factors and ways of avoiding them are appropriate nursing interventions for the anxiety. Interventions that teach alternative therapies to supplement drug therapy also give the patient some control over pain and are appropriate teaching regarding treatment of the headache. The other interventions may help to reduce anxiety generally but they do not address the etiologic factor of the anxiety.

How well did you know this?
1
Not at all
2
3
4
5
Perfectly
59
Q

The nurse is preparing to admit a newly diagnosed patient experiencing tonic-clonic seizures. What could the nurse delegate to unlicensed assistive personnel (UAP)?

a. Complete the admission assessment.
b. Explain the call system to the patient.
c. Obtain the suction equipment from the supply cabinet.
d. Place a padded tongue blade on the wall above the patient’s bed.

A

c. Obtain the suction equipment from the supply cabinet.

The unlicensed assistive personnel (UAP) is able to
obtain equipment from the supply cabinet or department. The RN may need to provide a list of necessary equipment and should set up the equipment and ensure proper functioning. The RN is responsible for the initial history and assessment as well as teaching the patient about the room’s call system. Padded tongue blades are no longer used and no effort should be made to place anything in the patient’s mouth during a seizure.

How well did you know this?
1
Not at all
2
3
4
5
Perfectly
60
Q

How do generalized seizures differ from focal seizures?

a. Focal seizures are confined to one side of the brain and remain focal in nature.
b. Generalized seizures result in loss of consciousness whereas focal seizures do not.
c. Generalized seizures result in temporary residual deficits during the postictal phase.
d. Generalized seizures have bilateral synchronous epileptic discharges affecting the whole brain at onset of the seizure.

A

d. Generalized seizures have bilateral synchronous epileptic discharges affecting the whole brain at onset of the seizure.

Generalized seizures have bilateral synchronous epileptic discharge affecting the entire brain at onset of the seizure. Loss of consciousness is also characteristic but many focal seizures also include an altered consciousness. Focal seizures begin in one side of the brain but may spread to involve the entire brain. Focal seizures that start with a local focus and spread to the entire brain, causing a secondary generalized seizure, are associated with a transient residual neurologic deficit postictally known as Todd’s paralysis.

How well did you know this?
1
Not at all
2
3
4
5
Perfectly
61
Q

Which type of seizure occurs in children, is also known as a petit mal seizure, and consists of a staring spell that lasts for a few seconds?

a. Atonic
b. Simple focal
c. Typical absence
d. Atypical absence

A

c. Typical absence

The typical absence seizure is also known as petit mal
and the child has staring spells that last for a few seconds. Atonic seizures occur when the patient falls from loss of muscle tone accompanied by brief unconsciousness. Simple focal seizures have focal motor, sensory, or autonomic symptoms related to the area of the brain involved without loss of consciousness. Staring spells in atypical absence seizures last longer than those in typical absence seizures and are accompanied by peculiar behavior during the seizure or confusion after the seizure.

How well did you know this?
1
Not at all
2
3
4
5
Perfectly
62
Q

The patient is diagnosed with complex focal seizures. Which characteristics are related to complex focal seizures (select all that apply)?

a. Formerly known as grand mal seizure
b. Often accompanied by incontinence or tongue or cheek biting
c. Psychomotor seizures with repetitive behaviors and lip smacking
d. Altered memory, sexual sensations, and distortions of visual or auditory sensations
e. Loss of consciousness and stiffening of the body with subsequent jerking of extremities
f. Often involves behavioral, emotional, and cognitive functions with altered consciousness

A

c. Psychomotor seizures with repetitive behaviors and lip smacking
d. Altered memory, sexual sensations, and distortions of visual or auditory
f. Often involves behavioral, emotional, and cognitive functions with altered consciousness

Complex focal seizures are psychomotor seizures with automatisms such as lip smacking. They cause altered consciousness or loss of consciousness producing a dreamlike state and may involve behavioral, emotional, or cognitive experiences without memory of what was done during the seizure. In generalized tonic-clonic seizures (previously known as grand mal seizures) there is loss of consciousness and stiffening of the body with subsequent jerking of extremities. Incontinence or tongue or cheek biting may also occur.

How well did you know this?
1
Not at all
2
3
4
5
Perfectly
63
Q

Which type of seizure is most likely to cause death for the patient?

a. Subclinical seizures
b. Myoclonic seizures
c. Psychogenic seizures
d. Tonic-clonic status epilepticus

A

d. Tonic-clonic status epilepticus

Tonic-clonic status epilepticus is most dangerous because the continuous seizing can cause respiratory insufficiency, hypoxemia, cardiac dysrhythmia, hyperthermia, and systemic acidosis, which can all be fatal. Subclinical seizures may occur in a patient who is sedated, so there is no physical movement. Myoclonic seizures may occur in clusters and have a sudden, excessive jerk of the body that may hurl the person to the ground. Psychogenic seizures are psychiatric in origin and diagnosed with videoelectroencephalography (EEG) monitoring. They occur in patients with a history of emotional abuse or a specific traumatic episode.

How well did you know this?
1
Not at all
2
3
4
5
Perfectly
64
Q

A patient admitted to the hospital following a generalized tonic-clonic seizure asks the nurse what caused the seizure. What is the best response by the nurse?

a. “So many factors can cause epilepsy that it is impossible to say what caused your seizure.”
b. “Epilepsy is an inherited disorder. Does anyone else in your family have a seizure disorder?”
c. “In seizures, some type of trigger causes sudden, abnormal bursts of electrical brain activity.”
d. “Scar tissue in the brain alters the chemical balance, creating uncontrolled electrical discharges.”

A

c. “In seizures, some type of trigger causes sudden, abnormal bursts of electrical brain activity.”

A seizure is a paroxysmal, uncontrolled discharge of neurons in the brain, which interrupts normal function, but the factor that causes the abnormal firing is not clear. Seizures may be precipitated by many factors and although scar tissue may make the brain neurons more likely to fire, it is not the usual cause of seizures. Epilepsy is established only by a pattern of spontaneous, recurring seizures.

How well did you know this?
1
Not at all
2
3
4
5
Perfectly
65
Q

A patient with a seizure disorder is being evaluated for surgical treatment of the seizures. The nurse recognizes that what is one of the requirements for surgical treatment?

a. Identification of scar tissue that is able to be removed
b. An adequate trial of drug therapy that had unsatisfactory results
c. Development of toxic syndromes from long-term use of antiseizure drugs
d. The presence of symptoms of cerebral degeneration from repeated seizures

A

b. An adequate trial of drug therapy that had unsatisfactory results

Most patients with seizure disorders maintain seizure control with medications but if surgery is considered, three requirements must be met: the diagnosis of epilepsy must be confirmed, there must have been an adequate trial with drug therapy without satisfactory results, and the electroclinical syndrome must be defined. The focal point must be localized but the presence of scar tissue is not required.

How well did you know this?
1
Not at all
2
3
4
5
Perfectly
66
Q

The nurse teaches the patient taking antiseizure drugs that this method is most commonly used to measure compliance and monitor for toxicity.

a. A daily seizure log
b. Urine testing for drug levels
c. Blood testing for drug levels
d. Monthly electroencephalography (EEG)

A

c. Blood testing for drug levels

Serum levels of antiseizure drugs are monitored regularly to maintain therapeutic levels of the drug, above which patients are likely to experience toxic effects and below which seizures are likely to occur. Many newer drugs do not require drug level monitoring because of large therapeutic ranges. A daily seizure log and urine testing for drug levels will not measure compliance or monitor for toxicity. EEGs have limited value in diagnosis of seizures and even less value in monitoring seizure control.

How well did you know this?
1
Not at all
2
3
4
5
Perfectly
67
Q

Priority Decision: When teaching a patient with a seizure disorder about the medication regimen, what is it most important for the nurse to emphasize?

a. The patient should increase the dosage of the medication if stress is increased.
b. Most over-the-counter and prescription drugs are safe to take with antiseizure drugs.
c. Stopping the medication abruptly may increase the intensity and frequency of seizures.
d. If gingival hypertrophy occurs, the drug should be stopped and the health care provider notified.

A

c. Stopping the medication abruptly may increase the intensity and frequency of seizures.

If antiseizure drugs are discontinued abruptly, seizures can be precipitated. Missed doses should be made up if the omission is remembered within 24 hours and patients should not adjust medications without professional guidance because this also can increase seizure frequency and may cause status epilepticus. Antiseizure drugs have numerous interactions with other drugs and the use of other medications should be evaluated by health professionals. If side effects occur, the physician should be notified and drug regimens evaluated.

How well did you know this?
1
Not at all
2
3
4
5
Perfectly
68
Q

Priority Decision: The nurse finds a patient in bed having a generalized tonic-clonic seizure. During the seizure activity, what actions should the nurse take (select all that apply)?

a. Loosen restrictive clothing.
b. Turn the patient to the side.
c. Protect the patient’s head from injury.
d. Place a padded tongue blade between the patient’s teeth.
e. Restrain the patient’s extremities to prevent soft tissue and bone injury.

A

a. Loosen restrictive clothing.
b. Turn the patient to the side.
c. Protect the patient’s head from injury.

The focus is on maintaining a patent airway and
preventing patient injury. The nurse should not place objects in the patient’s mouth or restrain the patient.

How well did you know this?
1
Not at all
2
3
4
5
Perfectly
69
Q

Following a generalized tonic-clonic seizure, the patient is tired and sleepy. What care should the nurse provide?

a. Suction the patient before allowing him to rest.
b. Allow the patient to sleep as long as he feels sleepy.
c. Stimulate the patient to increase his level of consciousness.
d. Check the patient’s level of consciousness every 15 minutes for an hour.

A

b. Allow the patient to sleep as long as he feels sleepy.

In the postictal phase of generalized tonic-clonic seizures, patients are usually very tired and may sleep for several hours and the nurse should allow the patient to sleep as long as necessary. Suctioning is performed only if needed and decreased level of consciousness is not a problem postictally unless a head injury has occurred during the seizure.

How well did you know this?
1
Not at all
2
3
4
5
Perfectly
70
Q

During the diagnosis and long-term management of a seizure disorder, what should the nurse recognize as one of the major needs of the patient?

a. Managing the complicated drug regimen of seizure control
b. Coping with the effects of negative social attitudes toward epilepsy
c. Adjusting to the very restricted lifestyle required by a diagnosis of epilepsy
d. Learning to minimize the effect of the condition in order to obtain employment

A

b. Coping with the effects of negative social attitudes toward epilepsy

One of the most common complications of a seizure disorder is the effect it has on the patient’s lifestyle. This is because of the social stigma attached to seizures, which causes patients to hide their diagnosis and to prefer not to be identified as having epilepsy. Medication regimens usually require only once- or twice-daily dosing and the major restrictions of lifestyle usually involve driving and high-risk environments. Job discrimination against the handicapped is prevented by federal and state laws and patients only need to identify their disease in case of medical emergencies.

How well did you know this?
1
Not at all
2
3
4
5
Perfectly
71
Q

A patient at the clinic for a routine health examination mentions that she is exhausted because her legs bother her so much at night that she cannot sleep. The nurse questions the patient further about her leg symptoms with what knowledge about restless legs syndrome?

a. The condition can be readily diagnosed with EMG.
b. Other more serious nervous system dysfunctions may be present.
c. Dopaminergic agents are often effective in managing the symptoms.
d. Symptoms can be controlled by vigorous exercise of the legs during the day.

A

c. Dopaminergic agents are often effective in managing the symptoms.

Restless legs syndrome that is not related to other pathologic processes, such as diabetes mellitus or rheumatic disorders, may be caused by a dysfunction in the basal ganglia circuits that use the neurotransmitter dopamine, which controls movements. Dopamine precursors and dopamine agonists, such as those used for parkinsonism, are effective in managing sensory and motor symptoms. Polysomnography studies during sleep are the only tests that have diagnostic value and although exercise should be encouraged, excessive leg exercise does not have an effect on the symptoms.

How well did you know this?
1
Not at all
2
3
4
5
Perfectly
72
Q

Which chronic neurologic disorder involves a deficiency of the neurotransmitters acetylcholine and γ aminobutyric acid (GABA) in the basal ganglia and extrapyramidal system?

a. Myasthenia gravis
b. Parkinson’s disease
c. Huntington’s disease
d. Amyotrophic lateral sclerosis (ALS)

A

c. Huntington’s disease

Huntington’s disease (HD) involves deficiency of
acetylcholine and γ-aminobutyric acid (GABA) in the basal ganglia and extrapyramidal system that causes the opposite symptoms of parkinsonism. Myasthenia gravis involves autoimmune antibody destruction of cholinergic receptors at the neuromuscular junction. Amyotrophic lateral sclerosis (ALS) involves degeneration of motor neurons in the brainstem and spinal cord.

How well did you know this?
1
Not at all
2
3
4
5
Perfectly
73
Q

A 38-year-old woman has newly diagnosed multiple sclerosis (MS) and asks the nurse what is going to happen to her. What is the best response by the nurse?

a. “You will have either periods of attacks and remissions or progression of nerve damage over time.”
b. “You need to plan for a continuous loss of movement, sensory functions, and mental capabilities.”
c. “You will most likely have a steady course of chronic progressive nerve damage that will change your
personality.”
d. “It is common for people with MS to have an acute attack of weakness and then not to have any other symptoms
for years.”

A

a. “You will have either periods of attacks and remissions or progression of nerve damage over time.”

Most patients with multiple sclerosis (MS) have
remissions and exacerbations of neurologic dysfunction or a relapsing-remitting initial course followed by progression with or without occasional relapses, minor remissions, and plateaus that progressively cause loss of motor, sensory, and cerebellar functions. Intellectual function generally remains intact but patients may experience anger, depression, or euphoria. A few people have chronic progressive deterioration and some may experience only occasional and mild symptoms for several years after onset.

How well did you know this?
1
Not at all
2
3
4
5
Perfectly
74
Q

During assessment of a patient admitted to the hospital with an acute exacerbation of MS, what should the nurse expect to find?

a. Tremors, dysphasia, and ptosis
b. Bowel and bladder incontinence and loss of memory
c. Motor impairment, visual disturbances, and paresthesias
d. Excessive involuntary movements, hearing loss, and ataxia

A

c. Motor impairment, visual disturbances, and paresthesias

Specific neurologic dysfunction of MS is caused by
destruction of myelin and replacement with glial scar tissue at specific areas in the nervous system. Motor, sensory, cerebellar, and emotional dysfunctions, including paresthesias as well as patchy blindness, blurred vision, pain radiating along the dermatome of the nerve, ataxia, and severe fatigue, are the most common manifestations of MS. Constipation and bladder dysfunctions, short-term memory loss, sexual dysfunction, anger, and depression or euphoria may also occur. Excessive involuntary movements and tremors are not seen in MS.

How well did you know this?
1
Not at all
2
3
4
5
Perfectly
75
Q

The nurse explains to a patient newly diagnosed with MS that the diagnosis is made primarily by

a. spinal x-ray findings.
b. T-cell analysis of the blood.
c. analysis of cerebrospinal fluid.
d. history and clinical manifestations.

A

d. history and clinical manifestations.

There is no specific diagnostic test for MS. A diagnosis is made primarily by history and clinical manifestations. Certain diagnostic tests may be used to help establish a diagnosis of MS. Positive findings on MRI include evidence of at least two inflammatory demyelinating lesions in at least two different locations within the central nervous system (CNS). Cerebrospinal fluid (CSF) may have increased immunoglobulin G and the presence of oligoclonal banding. Evoked potential responses are often delayed in persons with MS.

How well did you know this?
1
Not at all
2
3
4
5
Perfectly
76
Q

Mitoxantrone (Novantrone) is being considered as treatment for a patient with progressive-relapsing MS. The nurse explains that a disadvantage of this drug compared with other drugs used for MS is what?

a. It must be given subcutaneously every day.
b. It has a lifetime dose limit because of cardiac toxicity.
c. It is an anticholinergic agent that causes urinary incontinence.
d. It is an immunosuppressant agent that increases the risk for infection.

A

b. It has a lifetime dose limit because of cardiac toxicity.

Mitoxantrone (Novantrone) cannot be used for more than 2 to 3 years because it is an antineoplastic drug that causes cardiac toxicity, leukemia, and infertility. It is a monoclonal antibody given IV monthly when patients have inadequate responses to other drugs. It increases the risk of progressive multifocal leukoencephalopathy.

How well did you know this?
1
Not at all
2
3
4
5
Perfectly
77
Q

A patient with MS has a nursing diagnosis of self-care deficit related to muscle spasticity and neuromuscular deficits. In providing care for the patient, what is most important for the nurse to do?

a. Teach the family members how to care adequately for the patient’s needs.
b. Encourage the patient to maintain social interactions to prevent social isolation.
c. Promote the use of assistive devices so the patient can participate in self-care activities.
d. Perform all activities of daily living (ADLs) for the patient to conserve the patient’s energy.

A

c. Promote the use of assistive devices so the patient can participate in self-care activities.

The main goal in care of the patient with MS is to keep the patient active and maximally functional and promote self-care as much as possible to maintain independence. Assistive devices encourage independence while preserving the patient’s energy. No care activity that the patient can do for himself or herself should be performed by others. Involvement of the family in the patient’s care and maintenance of social interactions are also important but are not the priority in care.

How well did you know this?
1
Not at all
2
3
4
5
Perfectly
78
Q

A patient with newly diagnosed MS has been hospitalized for evaluation and initial treatment of the disease. Following discharge teaching, the nurse realizes that additional instruction is needed when the patient says what?

a. “It is important for me to avoid exposure to people with upper respiratory infections.”
b. “When I begin to feel better, I should stop taking the prednisone to prevent side effects.”
c. “I plan to use vitamin supplements and a high-protein diet to help manage my condition.”
d. “I must plan with my family how we are going to manage my care if I become more incapacitated.”

A

b. “When I begin to feel better, I should stop taking the prednisone to prevent side effects.”

Corticosteroids used in treating acute exacerbations of MS should not be abruptly stopped by the patient because adrenal insufficiency may result and prescribed tapering doses should be followed. Infections may exacerbate symptoms and should be avoided and high-protein diets with vitamin supplements are advocated. Long-term planning for increasing disability is also important.

How well did you know this?
1
Not at all
2
3
4
5
Perfectly
79
Q

The classic triad of manifestations associated with Parkinson’s disease is tremor, rigidity, and bradykinesia. What is a consequence related to rigidity?

a. Shuffling gait
b. Impaired handwriting
c. Lack of postural stability
d. Muscle soreness and pain

A

d. Muscle soreness and pain

The degeneration of dopamine-producing neurons in the substantia nigra of midbrain and basal ganglia lead to this triad of signs. Muscle soreness, pain, and slowness of movement are patient function consequences related to rigidity. Shuffling gait, lack of postural stability, absent arm swing while walking, absent blinking, masked facial expression, and difficulty initiating movement are all related to bradykinesia. Impaired handwriting and hand activities are related to the tremor of Parkinson’s disease (PD).

How well did you know this?
1
Not at all
2
3
4
5
Perfectly
80
Q

A patient with a tremor is being evaluated for Parkinson’s disease. The nurse explains to the patient that Parkinson’s disease can be confirmed by

a. CT and MRI scans.
b. relief of symptoms with administration of dopaminergic agents.
c. the presence of tremors that increase during voluntary movement.
d. cerebral angiogram that reveals the presence of cerebral atherosclerosis.

A

b. relief of symptoms with administration of dopaminergic agents.

Although clinical manifestations are characteristic in PD, no laboratory or diagnostic tests are specific for the condition. A diagnosis is made when at least two of the three signs of the classic triad are present and it is confirmed with a positive response to antiparkinsonian medication. Research regarding the role of genetic testing and MRI to diagnose PD is ongoing. Essential tremors increase during voluntary movement whereas the tremors of PD are more prominent at rest.

How well did you know this?
1
Not at all
2
3
4
5
Perfectly
81
Q

Which observation of the patient made by the nurse is most indicative of Parkinson’s disease?

a. Large, embellished handwriting
b. Weakness of one leg resulting in a limping walk
c. Difficulty rising from a chair and beginning to walk
d. Onset of muscle spasms occurring with voluntary movement

A

c. Difficulty rising from a chair and beginning to walk

The bradykinesia of PD prevents automatic movements and activities such as beginning to walk, rising from a chair, or even swallowing saliva cannot be executed unless they are consciously willed. Handwriting is affected by the tremor and results in the writing trailing off at the end of words. Specific limb weakness and muscle spasms are not characteristic of PD.

How well did you know this?
1
Not at all
2
3
4
5
Perfectly
82
Q

A patient with Parkinson’s disease is started on levodopa. What should the nurse explain about this drug?

a. It stimulates dopamine receptors in the basal ganglia.
b. It promotes the release of dopamine from brain neurons.
c. It is a precursor of dopamine that is converted to dopamine in the brain.
d. It prevents the excessive breakdown of dopamine in the peripheral tissues.

A

c. It is a precursor of dopamine that is converted to dopamine in the brain.

Peripheral dopamine does not cross the blood-brain barrier but its precursor, levodopa, is able to enter the brain, where it is converted to dopamine, increasing the supply that is deficient in PD. Other drugs used to treat PD include bromocriptine, which stimulates dopamine receptors in the basal ganglia, and amantadine, which blocks the reuptake of dopamine into presynaptic neurons. Carbidopa is an agent that is usually administered with levodopa to prevent the levodopa from being metabolized in peripheral tissues before it can reach the brain.

How well did you know this?
1
Not at all
2
3
4
5
Perfectly
83
Q

To reduce the risk for falls in the patient with Parkinson’s disease, what should the nurse teach the patient to do?

a. Use an elevated toilet seat.
b. Use a walker or cane for support.
c. Consciously lift the toes when stepping.
d. Rock side to side to initiate leg movements.

A

c. Consciously lift the toes when stepping.

The shuffling gait of PD causes the patient to be off
balance and at risk for falling. Teaching the patient to
use a wide stance with the feet apart, to lift the toes when walking, and to look ahead helps to promote a more balanced gait. Use of an elevated toilet seat and rocking from side to side will enable a patient to initiate movement. Canes and walkers are difficult for patients with PD to maneuver and may make the patient more prone to injury.

How well did you know this?
1
Not at all
2
3
4
5
Perfectly
84
Q

A patient with myasthenia gravis is admitted to the hospital with respiratory insufficiency and severe weakness. When is a diagnosis of cholinergic crisis made?

a. The patient’s respiration is impaired because of muscle weakness.
b. Administration of edrophonium (Tensilon) increases muscle weakness.
c. Administration of edrophonium (Tensilon) results in improved muscle contractility.
d. EMG reveals decreased response to repeated stimulation of muscles.

A

b. Administration of edrophonium (Tensilon) increases muscle weakness

The reduction of the acetylcholine (ACh) effect in
myasthenia gravis (MG) is treated with anticholinesterase drugs, which prolong the action of ACh at the neuromuscular synapse, but too much of these drugs will cause a cholinergic crisis with symptoms very similar to those of MG. To determine whether the patient's manifestations are due to a deficiency of ACh or to too much anticholinesterase drug, the anticholinesterase drug edrophonium chloride (Tensilon) is administered. If the patient is in cholinergic crisis, the patient's symptoms will worsen; if the patient is in a myasthenic crisis, the patient will improve.
How well did you know this?
1
Not at all
2
3
4
5
Perfectly
85
Q

During care of a patient in myasthenic crisis, maintenance of what is the nurse’s first priority for the patient?

a. Mobility
b. Nutrition
c. Respiratory function
d. Verbal communication

A

c. Respiratory function

The patient in myasthenic crisis has severe weakness and fatigability of all skeletal muscles, affecting the patient’s ability to breathe, swallow, talk, and move. However, the priority of nursing care is monitoring and maintaining adequate ventilation.

How well did you know this?
1
Not at all
2
3
4
5
Perfectly
86
Q

When providing care for a patient with ALS, the nurse recognizes what as one of the most distressing problems experienced by the patient?

a. Painful spasticity of the face and extremities
b. Retention of cognitive function with total degeneration of motor function
c. Uncontrollable writhing and twisting movements of the face, limbs, and body
d. Knowledge that there is a 50% chance the disease has been passed to any offspring

A

b. Retention of cognitive function with total degeneration of motor function

In ALS there is gradual degeneration of motor neurons with extreme muscle wasting from lack of stimulation and use. However, cognitive function is not impaired and patients feel trapped in a dying body. Chorea manifested by writhing, involuntary movements is characteristic of HD. As an autosomal dominant genetic disease, HD also has a 50% chance of being passed to each offspring.

How well did you know this?
1
Not at all
2
3
4
5
Perfectly
87
Q

In providing care for patients with chronic, progressive neurologic disease, what is the major goal of treatment that the nurse works toward?

a. Meet the patient’s personal care needs.
b. Return the patient to normal neurologic function.
c. Maximize neurologic functioning for as long as possible.
d. Prevent the development of additional chronic diseases.

A

c. Maximize neurologic functioning for as long as possible.

Many chronic neurologic diseases involve progressive deterioration in physical or mental capabilities and have no cure, with devastating results for patients and families. Health care providers can only attempt to alleviate physical symptoms, prevent complications, and assist patients in maximizing function and self-care abilities for as long as possible.

88
Q

A 73-year-old patient with a stroke experiences facial drooping on the right side and right-sided arm and leg paralysis. When admitting the patient, which clinical manifestation will the nurse expect to find?

a. Impulsive behavior
b. Right-sided neglect
c. Hyperactive left-sided tendon reflexes
d. Difficulty comprehending instructions

A

d. Difficulty comprehending instructions

Right-sided paralysis indicates a left-brain stroke, which will lead to difficulty with comprehension and use of language. The left-side reflexes are likely to be intact. Impulsive behavior and neglect are more likely with a right-side stroke.

89
Q

A female patient who had a stroke 24 hours ago has expressive aphasia. The nurse identifies the nursing diagnosis of impaired verbal communication. An appropriate nursing intervention to help the patient communicate is to

a. ask questions that the patient can answer with “yes” or “no.”
b. develop a list of words that the patient can read and practice reciting.
c. have the patient practice her facial and tongue exercises with a mirror.
d. prevent embarrassing the patient by answering for her if she does not respond.

A

a. ask questions that the patient can answer with “yes” or “no.”

Communication will be facilitated and less frustrating to the patient when questions that
require a “yes” or “no” response are used. When the language areas of the brain are injured,
the patient might not be able to read or recite words, which will frustrate the patient without
improving communication. Expressive aphasia is caused by damage to the language areas of
the brain, not by the areas that control the motor aspects of speech. The nurse should allow
time for the patient to respond.

90
Q

When family members ask the nurse about the purpose of the ventriculostomy system being used for intracranial pressure monitoring for a patient, which response by the nurse is best?

a. “This type of monitoring system is complex and highly skilled staff are needed.”
b. “The monitoring system helps show whether blood flow to the brain is adequate.”
c. “The ventriculostomy monitoring system helps check for alterations in cerebral perfusion pressure.”
d. “This monitoring system has multiple benefits including facilitation of cerebrospinal fluid drainage.”

A

b. “The monitoring system helps show whether blood flow to the brain is adequate.”

Short and simple explanations should be given to patients and family members. The other explanations are either too complicated to be easily understood or may increase the family member’s anxiety.

91
Q

A patient with a head injury has admission vital signs of blood pressure 128/68, pulse 110, and respirations 26. Which of these vital signs, if taken 1 hour after admission, will be of most concern to the nurse?

a. Blood pressure 156/60, pulse 55, respirations 12
b. Blood pressure 130/72, pulse 90, respirations 32
c. Blood pressure 148/78, pulse 112, respirations 28
d. Blood pressure 110/70, pulse 120, respirations 30

A

a. Blood pressure 156/60, pulse 55, respirations 12

Systolic hypertension with widening pulse pressure, bradycardia, and respiratory changes represent Cushing’s triad and indicate that the intracranial pressure (ICP) has increased, and brain herniation may be imminent unless immediate action is taken to reduce ICP. The other vital signs may indicate the need for changes in treatment, but they are not indicative of an immediately life-threatening process.

92
Q

When the nurse applies a painful stimulus to the nail beds of an unconscious patient, the patient responds with internal rotation, adduction, and flexion of the arms. The nurse documents this as

a. flexion withdrawal.
b. localization of pain.
c. decorticate posturing.
d. decerebrate posturing.

A

c. decorticate posturing.

Internal rotation, adduction, and flexion of the arms in an unconscious patient is documented as decorticate posturing. Extension of the arms and legs is decerebrate posturing. Because the flexion is generalized, it does not indicate localization of pain or flexion withdrawal.

93
Q

Which parameter is best for the nurse to monitor to determine whether the prescribed IV mannitol (Osmitrol) has been effective for an unconscious patient?

a. Hematocrit
b. Blood pressure
c. Oxygen saturation
d. Intracranial pressure

A

d. Intracranial pressure

Mannitol is an osmotic diuretic and will reduce cerebral edema and intracranial pressure. It may initially reduce hematocrit and increase blood pressure, but these are not the best parameters for evaluation of the effectiveness of the drug. Oxygen saturation will not directly improve as a result of mannitol administration.

94
Q

A patient with a head injury opens the eyes to verbal stimulation, curses when stimulated, and does not respond to a verbal command to move but attempts to remove a painful stimulus. The nurse records the patient’s Glasgow Coma Scale score as

a. 9.
b. 11.
c. 13.
d. 15.

A

b. 11.

The patient has a score of 3 for eye opening, 3 for best verbal response, and 5 for best motor response.

95
Q

Following a head injury, an unconscious 32-year-old patient is admitted to the emergency department (ED). The patient’s spouse and children stay at the patient’s side and constantly ask about the treatment being given. What action is best for the nurse to take?

a. Ask the family to stay in the waiting room until the initial assessment is completed.
b. Allow the family to stay with the patient and briefly explain all procedures to them.
c. Call the family’s pastor or spiritual advisor to support them while initial care is given.
d. Refer the family members to the hospital counseling service to deal with their anxiety.

A

b. Allow the family to stay with the patient and briefly explain all procedures to them.

The need for information about the diagnosis and care is very high in family members of acutely ill patients, and the nurse should allow the family to observe care and explain the procedures. A pastor or counseling service can offer some support, but research supports information as being more effective. Asking the family to stay in the waiting room will increase their anxiety.

96
Q

An unconscious patient has a nursing diagnosis of ineffective cerebral tissue perfusion related to cerebral tissue swelling. Which nursing intervention will be included in the plan of care?

a. Keep the head of the bed elevated to 30 degrees.
b. Position the patient with the knees and hips flexed.
c. Encourage coughing and deep breathing to improve oxygenation.
d. Cluster nursing interventions to provide uninterrupted rest periods.

A

a. Keep the head of the bed elevated to 30 degrees.

The patient with increased intracranial pressure (ICP) should be maintained in the head-up position to help reduce ICP. Flexion of the hips and knees increases abdominal pressure, which increases ICP. Because the stimulation associated with nursing interventions increases ICP, clustering interventions will progressively elevate ICP. Coughing increases intrathoracic pressure and ICP.

97
Q

After noting that a patient with a head injury has clear nasal drainage, which action should the nurse take?

a. Have the patient blow the nose.
b. Check the nasal drainage for glucose.
c. Assure the patient that rhinorrhea is normal after a head injury.
d. Obtain a specimen of the fluid to send for culture and sensitivity.

A

b. Check the nasal drainage for glucose.

Clear nasal drainage in a patient with a head injury suggests a dural tear and cerebrospinal fluid (CSF) leakage. If the drainage is CSF, it will test positive for glucose. Fluid leaking from the nose will have normal nasal flora, so culture and sensitivity will not be useful. Blowing the nose is avoided to prevent CSF leakage.

98
Q

A patient who has a head injury is diagnosed with a concussion. Which action will the nurse plan to take?

a. Coordinate the transfer of the patient to the operating room.
b. Provide discharge instructions about monitoring neurologic status.
c. Transport the patient to radiology for magnetic resonance imaging (MRI) of the brain.
d. Arrange to admit the patient to the neurologic unit for observation for 24 hours.

A

b. Provide discharge instructions about monitoring neurologic status.

A patient with a minor head trauma is usually discharged with instructions about neurologic monitoring and the need to return if neurologic status deteriorates. MRI, hospital admission, or surgery are not indicated in a patient with a concussion.

99
Q

A patient who is suspected of having an epidural hematoma is admitted to the emergency department. Which action will the nurse plan to take?

a. Administer IV furosemide (Lasix).
b. Initiate high-dose barbiturate therapy.
c. Type and crossmatch for blood transfusion.
d. Prepare the patient for immediate craniotomy.

A

d. Prepare the patient for immediate craniotomy.

The principal treatment for epidural hematoma is rapid surgery to remove the hematoma and prevent herniation. If intracranial pressure (ICP) is elevated after surgery, furosemide or high-dose barbiturate therapy may be needed, but these will not be of benefit unless the hematoma is removed. Minimal blood loss occurs with head injuries, and transfusion is usually not necessary.

100
Q

While admitting a patient with a basal skull fracture, the nurse notes clear drainage from the patient’s nose. Which of these admission orders should the nurse question?

a. Insert nasogastric tube.
b. Turn patient every 2 hours.
c. Keep the head of bed elevated.
d. Apply cold packs for facial bruising.

A

a. Insert nasogastric tube.

Rhinorrhea may indicate a dural tear with cerebrospinal fluid (CSF) leakage, and insertion of a nasogastric tube will increase the risk for infections such as meningitis. Turning the patient, elevating the head, and applying cold pack are appropriate orders.

101
Q

Which assessment information will the nurse collect to determine whether a patient is developing postconcussion syndrome?

a. Muscle resistance
b. Short-term memory
c. Glasgow coma scale
d. Pupil reaction to light

A

b. Short-term memory

Decreased short-term memory is one indication of postconcussion syndrome. The other data may be assessed but are not indications of postconcussion syndrome.

102
Q

When admitting a patient who has a tumor of the right frontal lobe, the nurse would expect to find

a. judgment changes.
b. expressive aphasia.
c. right-sided weakness.
d. difficulty swallowing.

A

a. judgment changes.

The frontal lobes control intellectual activities such as judgment. Speech is controlled in the parietal lobe. Weakness and hemiplegia occur on the contralateral side from the tumor. Swallowing is controlled by the brainstem.

103
Q

Which statement by a patient who is being discharged from the emergency department (ED) after a head injury indicates a need for intervention by the nurse?

a. “I will return if I feel dizzy or nauseated.”
b. “I am going to drive home and go to bed.”
c. “I do not even remember being in an accident.”
d. “I can take acetaminophen (Tylenol) for my headache.”

A

b. “I am going to drive home and go to bed.”

Following a head injury, the patient should avoid operating heavy machinery. Retrograde amnesia is common after a concussion. The patient can take acetaminophen for headache and should return if symptoms of increased intracranial pressure such as dizziness or nausea occur.

104
Q

After having a craniectomy and left anterior fossae incision, a patient has a nursing diagnosis of impaired physical mobility related to decreased level of consciousness and weakness. An appropriate nursing intervention is to

a. position the bed flat and log roll the patient.
b. cluster nursing activities to allow longer rest periods.
c. turn and reposition the patient side to side every 2 hours.
d. perform range-of-motion (ROM) exercises every 4 hours.

A

d. perform range-of-motion (ROM) exercises every 4 hours.

ROM exercises will help to prevent the complications of immobility. Patients with anterior craniotomies are positioned with the head elevated. The patient with a craniectomy should not be turned to the operative side. When the patient is weak, clustering nursing activities may lead to more fatigue and weakness.

105
Q

A patient who has bacterial meningitis is disoriented and anxious. Which nursing action will be included in the plan of care?

a. Encourage family members to remain at the bedside.
b. Apply soft restraints to protect the patient from injury.
c. Keep the room well-lighted to improve patient orientation.
d. Minimize contact with the patient to decrease sensory input.

A

a. Encourage family members to remain at the bedside.

Patients with meningitis and disorientation will be calmed by the presence of someone familiar at the bedside. Restraints should be avoided because they increase agitation and anxiety. The patient requires frequent assessment for complications; the use of touch and a soothing voice will decrease anxiety for most patients. The patient will have photophobia, so the light should be dim.

106
Q

The community health nurse is developing a program to decrease the incidence of meningitis in adolescents and young adults. Which nursing action is most important?

a. Vaccinate 11- and 12-year-old children against Haemophilus influenzae.
b. Emphasize the importance of hand washing to prevent spread of infection.
c. Immunize adolescents and college freshman against Neisseria meningitides.
d. Encourage adolescents and young adults to avoid crowded areas in the winter.

A

c. Immunize adolescents and college freshman against Neisseria meningitides.

The Neisseria meningitides vaccination is recommended for children ages 11 and 12, unvaccinated teens entering high school, and college freshmen. Hand washing may help decrease the spread of bacteria, but it is not as effective as immunization. Vaccination with Haemophilus influenzae is for infants and toddlers. Because adolescents and young adults are in school or the workplace, avoiding crowds is not realistic.

107
Q

While caring for a patient who has just been admitted with meningococcal meningitis, the RN observes all of the following. Which one requires action by the RN?

a. The bedrails at the head and foot of the bed are both elevated.
b. The patient receives a regular diet from the dietary department.
c. The nursing assistant goes into the patient’s room without a mask.
d. The lights in the patient’s room are turned off and the blinds are shut.

A

c. The nursing assistant goes into the patient’s room without a mask.

Meningococcal meningitis is spread by respiratory secretions, so it is important to maintain respiratory isolation as well as standard precautions. Because the patient may be confused and weak, bedrails should be elevated at both the food and head of the bed. Low light levels in the room decrease pain caused by photophobia. Nutrition is an important aspect of care in a patient with meningitis.

108
Q

When assessing a patient with bacterial meningitis, the nurse obtains the following data. Which finding should be reported immediately to the health care provider?

a. The patient has a positive Kernig’s sign.
b. The patient complains of having a stiff neck.
c. The patient’s temperature is 101° F (38.3° C).
d. The patient’s blood pressure is 86/42 mm Hg.

A

d. The patient’s blood pressure is 86/42 mm Hg.

Shock is a serious complication of meningitis, and the patient’s low blood pressure indicates the need for interventions such as fluids or vasopressors. Nuchal rigidity and a positive Kernig’s sign are expected with bacterial meningitis. The nurse should intervene to lower the temperature, but this is not as life threatening as the hypotension.

109
Q

A patient has a systemic BP of 108/51 mm Hg and an intracranial pressure (ICP) of 14 mm Hg. Which action should the nurse take first?

a. Elevate the head of the patient’s bed to 60 degrees.
b. Document the BP and ICP in the patient’s record.
c. Report the BP and ICP to the health care provider.
d. Continue to monitor the patient’s vital signs and ICP.

A

c. Report the BP and ICP to the health care provider.

The patient’s cerebral perfusion pressure is 56 mm Hg, below the normal of 60 to 100 mm Hg and approaching the level of ischemia and neuronal death. Immediate changes in the patient’s therapy such as fluid infusion or vasopressor administration are needed to improve the cerebral perfusion pressure. Adjustments in the head elevation should only be done after consulting with the health care provider. Continued monitoring and documentation also will be done, but they are not the first actions that the nurse should take.

110
Q

After suctioning, the nurse notes that the intracranial pressure for a patient with a traumatic head injury has increased from 14 to 16 mm Hg. Which action should the nurse take first?

a. Document the increase in intracranial pressure.
b. Assure that the patient’s neck is not in a flexed position.
c. Notify the health care provider about the change in pressure.
d. Increase the rate of the prescribed propofol (Diprovan) infusion.

A

b. Assure that the patient’s neck is not in a flexed position.

Since suctioning will cause a transient increase in intracranial pressure, the nurse should initially check for other factors that might be contributing to the increase and observe the patient for a few minutes. Documentation is needed, but this is not the first action. There is no need to notify the health care provider about this expected reaction to suctioning. Propofol is used to control patient anxiety or agitation; there is no indication that anxiety has contributed to the increase in intracranial pressure.

111
Q

Which of these patients is most appropriate for the intensive care unit (ICU) charge nurse to assign to an RN who has floated from the medical unit?

a. A 44-year-old receiving IV antibiotics for meningococcal meningitis
b. A 23-year-old who had a skull fracture and craniotomy the previous day
c. A 30-year-old who has an intracranial pressure (ICP) monitor in place after a head injury a week ago
d. A 61-year-old who has increased ICP and is receiving hyperventilation therapy

A

a. A 44-year-old receiving IV antibiotics for meningococcal meningitis

An RN who works on a medical unit will be familiar with administration of IV antibiotics and with meningitis. The postcraniotomy patient, patient with an ICP monitor, and the patient on a ventilator should be assigned to an RN familiar with the care of critically ill patients.

112
Q

A patient with possible cerebral edema has a serum sodium level of 115 mEq/L (115 mmol/L) and a decreasing level of consciousness (LOC) and complains of a headache. Which of these prescribed interventions should the nurse implement first?

a. Draw blood for arterial blood gases (ABGs).
b. Administer 5% hypertonic saline intravenously.
c. Administer acetaminophen (Tylenol) 650 mg orally.
d. Send patient for computed tomography (CT) of the head.

A

b. Administer 5% hypertonic saline intravenously.

The patient’s low sodium indicates that hyponatremia may be causing the cerebral edema, and the nurse’s first action should be to correct the low sodium level. Acetaminophen (Tylenol) will have minimal effect on the headache because it is caused by cerebral edema and increased intra-cranial pressure (ICP). Drawing ABGs and obtaining a CT scan may add some useful information, but the low sodium level may lead to seizures unless it is addressed quickly.

113
Q

After the emergency department nurse has received a status report on the following patients who have been admitted with head injuries, which patient should the nurse assess first?

a. A patient whose cranial x-ray shows a linear skull fracture
b. A patient who has an initial Glasgow Coma Scale score of 13
c. A patient who lost consciousness for a few seconds after a fall
d. A patient whose right pupil is 10 mm and unresponsive to light

A

d. A patient whose right pupil is 10 mm and unresponsive to light

The dilated and nonresponsive pupil may indicate an intracerebral hemorrhage and increased intracranial pressure. The other patients are not at immediate risk for complications such as herniation.

114
Q

Which assessment finding in a patient who was admitted the previous day with a basilar skull fracture is most important to report to the health care provider?

a. Bruising under both eyes
b. Complaint of severe headache
c. Large ecchymosis behind one ear
d. Temperature of 101.5° F (38.6° C)

A

d. Temperature of 101.5° F (38.6° C)

Patients who have basilar skull fractures are at risk for meningitis, so the elevated temperature should be reported to the health care provider. The other findings are typical of a patient with a basilar skull fracture.

115
Q

When a patient’s intracranial pressure (ICP) is being monitored with an intraventricular catheter, which information obtained by the nurse is most important to communicate to the health care provider?

a. Oral temperature 101.6° F
b. Apical pulse 102 beats/min
c. Intracranial pressure 15 mm Hg
d. Mean arterial pressure 90 mm Hg

A

a. Oral temperature 101.6° F

Infection is a serious consideration with ICP monitoring, especially with intraventricular catheters. The temperature indicates the need for antibiotics or removal of the monitor. The ICP, arterial pressure, and apical pulse are all borderline high but require only ongoing monitoring at this time.

116
Q

The charge nurse observes an inexperienced staff nurse who is caring for a patient who has had a craniotomy for a brain tumor. Which action by the inexperienced nurse requires the charge nurse to intervene?

a. The staff nurse suctions the patient every 2 hours.
b. The staff nurse assesses neurologic status every hour.
c. The staff nurse elevates the head of the bed to 30 degrees.
d. The staff nurse administers a mild analgesic before turning the patient.

A

a. The staff nurse suctions the patient every 2 hours.

Suctioning increases intracranial pressure and is done only when the patient’s respiratory condition indicates it is needed. The other actions by the staff nurse are appropriate.

117
Q

A patient is brought to the emergency department (ED) by ambulance after being found unconscious on the bathroom floor by the spouse. Which action will the nurse take first?

a. Obtain oxygen saturation.
b. Check pupil reaction to light.
c. Palpate the head for hematoma.
d. Assess Glasgow Coma Scale (GCS).

A

a. Obtain oxygen saturation.

Airway patency and breathing are the most vital functions and should be assessed first. The neurologic assessments should be accomplished next and the health and medication history last.

118
Q

The care plan for a patient who has increased intracranial pressure and a ventriculostomy includes the following nursing actions. Which action can the nurse delegate to nursing assistive personnel (NAP) who regularly work in the intensive care unit?

a. Monitor cerebrospinal fluid color hourly.
b. Document intracranial pressure every hour.
c. Turn and reposition the patient every 2 hours.
d. Check capillary blood glucose level every 6 hours.

A

d. Check capillary blood glucose level every 6 hours.

Experienced NAP can obtain capillary blood glucose levels when they have been trained and evaluated in the skill. Monitoring and documentation of cerebrospinal fluid (CSF) color and intracranial pressure (ICP) require RN-level education and scope of practice. Although repositioning patients is frequently delegated to NAP, repositioning a patient with a ventriculostomy is complex and should be done by the RN.

119
Q

Which information about a patient who is hospitalized after a traumatic brain injury requires the most rapid action by the nurse?

a. Intracranial pressure of 15 mm Hg
b. Cerebrospinal fluid (CSF) drainage of 15 mL/hour
c. Pressure of oxygen in brain tissue (PbtO2) is 14 mm Hg
d. Cardiac monitor shows sinus tachycardia, with a heart rate of 126 beats/min

A

c. Pressure of oxygen in brain tissue (PbtO2) is 14 mm Hg

The PbtO2 should be 20 to 40 mm Hg. Lower levels indicate brain ischemia. An intracranial pressure (ICP) of 15 mm Hg is at the upper limit of normal. CSF is produced at a rate of 20 to 30 mL/hour. The reason for the sinus tachycardia should be investigated, but the elevated heart rate is not as concerning as the decrease in PbtO2.

120
Q

When caring for a patient who has had a head injury, which assessment information requires the most rapid action by the nurse?

a. The patient is more difficult to arouse.
b. The patient’s pulse is slightly irregular.
c. The patient’s blood pressure increases from 120/54 to 136/62 mm Hg.
d. The patient complains of a headache at pain level 5 of a 10-point scale.

A

a. The patient is more difficult to arouse.

The change in level of consciousness (LOC) is an indicator of increased intracranial pressure (ICP) and suggests that action by the nurse is needed to prevent complications. The change in BP should be monitored but is not an indicator of a need for immediate nursing action. Headache is not unusual in a patient after a head injury. A slightly irregular apical pulse is not unusual.

121
Q

The nurse obtains these assessment findings for a patient who has a head injury. Which finding should be reported rapidly to the health care provider?

a. Urine output of 800 mL in the last hour
b. Intracranial pressure of 16 mm Hg when patient is turned
c. Ventriculostomy drains 10 mL of cerebrospinal fluid per hour
d. LICOX brain tissue oxygenation catheter shows PbtO2 of 38 mm Hg

A

a. Urine output of 800 mL in the last hour

The high urine output indicates that diabetes insipidus may be developing and interventions to prevent dehydration need to be rapidly implemented. The other data do not indicate a need for any change in therapy.

122
Q

When admitting a patient with a possible coup-contracoup injury after a car accident to the emergency department, the nurse obtains the following information. Which finding is most important to report to the health care provider?

a. The patient takes warfarin (Coumadin) daily.
b. The patient’s blood pressure is 162/94 mm Hg.
c. The patient is unable to remember the accident.
d. The patient complains of a severe dull headache.

A

a. The patient takes warfarin (Coumadin) daily.

The use of anticoagulants increases the risk for intracranial hemorrhage and should be immediately reported. The other information would not be unusual in a patient with a head injury who had just arrived to the ED.

123
Q

A patient admitted with bacterial meningitis and a temperature of 102° F (38.8° C) has orders for all of these collaborative interventions. Which action should the nurse take first?

a. Administer ceftizoxime (Cefizox) 1 g IV.
b. Use a cooling blanket to lower temperature.
c. Swap the nasopharyngeal mucosa for cultures.
d. Give acetaminophen (Tylenol) 650 mg PO.

A

c. Swap the nasopharyngeal mucosa for cultures.

Antibiotic therapy should be instituted rapidly in bacterial meningitis, but cultures must be done before antibiotics are started. As soon as the cultures are done, the antibiotic should be started. Hypothermia therapy and acetaminophen administration are appropriate but can be started after the other actions are implemented.

124
Q

The earliest signs of increased ICP the nurse should assess for include

a. Cushing’s triad
b. unexpected vomiting
c. decreasing level of consciousness (LOC)
d. dilated pupil with sluggish response to light

A

c. decreasing level of consciousness (LOC)

One of the most sensitive signs of increased intracranial pressure (ICP) is a decreasing LOC. A decrease in LOC will occur before changes in vital signs, ocular signs, and projectile vomiting occur

125
Q

The nurse recognizes the presence of Cushing’s triad in the patient with

a. Increased pulse, irregular respiration, increased BP
b. decreased pulse, irregular respiration, increased pulse pressure
c. increased pulse, decreased respiration, increased pulse pressure
d. decreased pulse, increased respiration, decreased systolic BP

A

b. decreased pulse, irregular respiration, increased pulse pressure

Cushing’s triad consists of three vital sign measures that reflect ICP and its effect on the medulla, the hypothalamus, the pons, and the thalamus. Because these structures are very deep, Cushing’s triad is usually a late sign of ICP. The signs include an increasing systolic BP with a widening pulse pressure, a bradycardia with a full and bounding pulse, and irregular respirations.

126
Q

Increased ICP in the left cerebral cortex, caused by intracranial bleeding causes displacement of brain tissue to the right hemisphere beneath the falx cerebri. The nurse knows that this is referred to as

a. uncal herniation
b. tentorial herniation
c. cingulate herniation
d. temporal lobe herniation

A

c. cingulate herniation

Cingulate herniation- the dural structures that separate the two hemispheres and the cerebral hemispheres from the cerebellum influence the patterns of cerebral herniation. A cingulated herniation occurs where there is lateral displacement of brain tissue beneath the falx cerebri.

127
Q

A patient has ICP monitoring with an intraventricular catheter. A priority nursing intervention for the patient is

a. aseptic technique to prevent infection
b. constant monitoring of ICP waveforms
c. removal of CSF to maintain normal ICP
d. sampling CSF to determine abnormalities

A

a. aseptic technique to prevent infection

Aseptic technique to prevent infection- An intraventricular catheter is a fluid coupled system that can provide direct access for microorganisms to enter the ventricles of the brain, and aseptic technique is a very high nursing priority to decrease the risk for infection. Constant monitoring of ICP waveforms is not usually necessary, and removal of CSF for sampling or to maintain normal ICP is done only when specifically ordered

128
Q

Metabolic and nutritional needs of the patient with increased ICP are best met with

a. enteral feedings that are low in sodium
b. the simple glucose available in D5W IV solutions
c. a fluid restriction that promotes a moderate dehydration
d. balanced, essential nutrition in a form that the patient can tolerate

A

d. balanced, essential nutrition in a form that the patient can tolerate

Balanced, essential nutrition in a form that the patient can tolerate= A patient with increased ICP is in a hypermetabolic and hypercatabolic state and needs adequate glucose to maintain fuel for the brain and other nutrients to meet metabolic needs. Malnutrition promotes cerebral edema, and if a patient cannot take oral nutrition, other means of providing nutrition should be used, such as tube feedings or parenteral nutrition. Glucose alone is not adequate to meet nutritional requirements, and 5% dextrose solutions may increase cerebral edema by lowering serum osmolarity. Patients should remain in a normovolemic fluid state with close monitoring of clinical factors such as urine output, fluid intake, serum and urine osmolality, serum electrolytes, and insensible losses.

129
Q

A patient with an intracranial problem does not open his eyes to any stimulus, has no verbal response except moaning and muttering when stimulated, and flexes his arm in response to painful stimuli. The nurse records the patients GCS score as

a. 6
b. 7
c. 9
d. 11

A

B. 7

  • no opening of eyes = 1; incomprehensible words= 2, flexion withdrawal = 4
    Total = 7
130
Q

When assessing the body function of a patient with increased ICP, the nurse should initially assess

a. corneal reflex testing
b. extremity strength testing
c. pupillary reaction to light
d. circulatory and respiratory status

A

d. circulatory and respiratory status

Of the body functions that should be assessed in an unconscious patient, cardiopulmonary status is the most vital function and gives priorities to the ABCs (airway, breathing, and circulation)

131
Q

CN III originating in the midbrain is assessed by the nurse for an early indication of pressure on the brainstem by

a. assessing for nystagmus
b. testing the corneal reflex
c. testing pupillary reaction to light
d. testing for oculocephalic (doll’s eye) reflex

A

C. Testing pupillary reaction to light

One of the functions of CN III, the oculomotor nerve, is pupillary constriction, and testing for pupillary constriction is important to identify patients at risk for brainstem herniation caused by increased ICP. The corneal reflex is used to assess the functions of CN V and VII, and the oculocephalic reflex tests all cranial nerves involved with eye movement. Nystagmus is commonly associatted with specific lesions or chemical toxicities and is not a definitive sign of ICP

132
Q

A patient has a nursing diagnosis of risk for ineffective cerebral tissue perfusion related to cerebral edema. An appropriate nursing intervention for the patient is

a. avoiding positioning the patient with neck and hip flexion
b. maintaining hyperventilation to a PaCO2 of 15 to 20 mm Hg
c. clustering nursing activities to provide periods of uninterrupted rest
d. routine suctioning to prevent accumulation of respiratory secretions

A

A. Avoiding positioning the patient with neck and hip flexion

Nursing care activities that increase ICP include hip and neck flexion, suctioning, clustering care activities, and noxious stimuli; they should be avoided or performed as little as possible in the patient with increased ICP. Lowering the PaCO2 below 20 mm Hg can cause ischemia and worsening of ICP; the PaCO2 should be maintained at 30 to 35 mm Hg.

133
Q

An unconscious patient with increased ICP in on ventilatory support. The nurse notifies the health care provider when arterial blood gas measurement results reveal a

a. pH of 7.43
b. SaO2 of 94%
c. PaO2 of 50 mm Hg
d. PaCO2 of 30 mm Hg

A

C. PaO2 of 50 mm Hg

A PaO2 of 50 mm Hg reflects a hypoxemia that may lead to further decreased cerebral perfusion and hypoxia and must be corrected. The pH of SaO2 are within normal range, and a PaCO2 of 30 mm Hg reflects acceptable value for the patient with increased ICP

134
Q

The nurse is monitoring a patient for increased ICP following a head injury. Which of the following manifestations indicate an increased ICP (select all that apply)

a. fever
b. oriented to name only
c. narrowing pulse pressure
d. dilated right pupil > left pupil
e. decorticate posturing to painful stimulus

A

a. fever
b. oriented to name only
d. dilated right pupil > left pupil
e. decorticate posturing to painful stimulus

The first sign of increased ICP is a change in LOC. Other manifestations are dilated ipsilateral pupil, changes in motor response such as posturing, and fever, which may indicate pressure on the hypothalamus. Changes in vital signs would be an increased systolic BP with widened pulse pressure and bradycardia

135
Q

While the nurse performs ROM on an unconscious patient with increased ICP, the patient experiences severe decerebrate posturing reflexes. The nurse should

a. use restraints to protect the patient from injury
b. administer CNS depressants to lightly sedate the patient
c. perform the exercises less frequently because posturing can increase ICP
d. continue the exercises because they are necessary to maintain musculoskeletal function

A

C. Perform the exercises less frequently because posturing can increase ICP

If reflex posturing occurs during ROM or positioning of the patient, these activities should be done less frequently until the patient’s condition stabilizes, because posturing can case increases in ICP. Neither restraints nor CNS depressants would be indicated.

136
Q

A patient with a head injury has bloody drainage from the ear. To determine whether CSF is present in the drainage, the nurse

a. examines the tympanic membrane for a tear
b. tests the fluid for a halo sign on a white dressing
c. tests the fluid with a glucose identifying strip or stick
d. collects 5 mL of fluid in a test tube and sends it to the laboratory for analysis

A

B. Tests the fluid for a halo sing on a white dressing

Testing clear drainage for CSF in nasal or ear drainage may be done with a Dextrostik or Tes-Tape strip, but if blood is present, the glucose in the blood will produce and unreliable result. To test bloody drainage, the nurse should test the fluid for a halo or ring that occurs when a yellowish ring encircles blood dripped onto a white pad or towel

137
Q

The nurse suspects the presence of an arterial epidural hematoma in the patient who experiences

a. failure to regain consciousness following a head injury
b. a rapid deterioration of neurologic function within 24 to 48 hours following a head injury
c. nonspecific, nonlocalizing progression of alteration in LOC occurring over weeks or months
d. unconsciousness at the time of a head injury with a brief period of consciousness followed by a decrease in LOC

A

d. unconsciousness at the time of a head injury with a brief period of consciousness followed by a decrease in LOC

An arterial epidural hematoma is the most acute neurologic emergency, and the typical symptoms include unconsciousness at the scene, with a brief lucid interval followed by a decrease in LOC. An acute subdural hematoma manifests signs within 48 hours of an injury; a chronic subdural hematoma develops over weeks or months

138
Q

Skull radiographs and a computed tomography (CT) scan provide evidence of a depressed parietal fracture with a subdural hematoma in a patient admitted to the emergency department following an automobile accident. In planning care for the patient, the nurse anticipates that

a. the patient will receive life-support measures until the condition stabilizes
b. immediate burr holes will be made to rapidly decompress the intracranial activity
c. the patient will be treated conservatively with close monitoring for changes in neurologic condition
d. the patient will be taken to surgery for a craniotomy for evacuation of blood and decompression of the cranium

A

d. the patient will be taken to surgery for a craniotomy for evacuation of blood and decompression of the cranium

When there is a depressed fracture and fractures with loose fragments, a craniotomy is indicated to elevate the depressed bone and remove free fragments. A craniotomy is also indicated in cases of acute subdural and epidural hematomas to remove the blood and control the bleeding. Burr holes may be used in an extreme emergency for rapid decompression, but with a depressed fracture, surgery would be the treatment of choice

139
Q

When a patient is admitted to the emergency department following a head injury, the nurse’s first priority in management of the patient once a patent airway is confirmed is

a. maintaining cervical spine precautions
b. determining the presence of increased ICP
c. monitoring for changes in neurologic status
d. establishing IV access with a large-bore catheter

A

a. maintaining cervical spine precautions

In addition to monitoring for a patent airway during emergency care of the patient with a head injury, the nurse must always assume that a patient with a head injury may have a cervical spine injury. Maintaining cervical spine precautions in all assessment and treatment activities with the patient is essential to prevent additional neurologic damage.

140
Q

A 54-year old man is recovering from a skull fracture with a subacute subdural hematoma. He has return of motor control and orientation but appears apathetic and has reduced awareness of his environment. When planning discharge for the patient, the nurse explains to the patient and the family that

a. continuous improvement in the patient’s condition should occur until he has returned to pre trauma status
b. the patient’s complete recovery may take years, and the family should plan for his long term dependent care
c. the patient is likely to have long term emotional and mental changes that may require continued professional help
d. role changes in family members will be necessary because the patient will be dependent on his family for care and support

A

c. the patient is likely to have long term emotional and mental changes that may require continued professional help

Residual mental and emotional changes of brain trauma with personality changes are often the most incapacitating problems following head injury and are common in patients who have been comatose longer than 6 hours. Families must be prepared for changes in the patient’s behavior to avoid family-patient friction and maintain family functioning, and professional assistance may be required. There is no indication he will be dependent on others for care, but he likely will not return to pre trauma status

141
Q

Assisting the family to understand what is happening to the patient is an especially important role of the nurse when the patient has a tumor of the

a. ventricles
b. frontal lobe
c. parietal lobe
d. occipital lobe

A

B. Frontal lobe

frontal lobe tumors often lead to loss of emotional control, confusion, memory loss, disorientation, and personality changes that are very disturbing and frightening to the family. Physical symptoms, such as blindness, disturbances in sensation and perception, and even seizures, that occur with other tumors are more likely to be understood and accepted by the family

142
Q

For the patient undergoing a craniotomy, the nurse provides information about the use of wigs and hairpieces or other methods to disguise hair loss

a. during pre operative teaching
b. in the patient asks about their use
c. in the immediate postoperative period
d. when the patient expresses negative feelings about his or her appearance

A

A. During pre operative teaching

The prevent undue concern and anxiety about hair loss and postoperative self-esteem disturbances, a patient undergoing cranial surgery should be informed pre operatively that the head is usually shaved in surgery while the patient is anesthetized and that methods can be used after the dressings are removed postoperatively to disguise the hair loss. In the immediate postoperative period, the patient is very ill, and the focus is on maintaining neurologic function, bur preoperatively the nurse should anticipate the patient’s postoperative need for self-esteem and maintenance of appearance.

143
Q

Successful achievement of patient outcomes for the patient with cranial surgery would be best indicated by the

a. ability to return home in 6 days
b. ability to meet all self-care needs
c. acceptance of residual neurologic deficits
d. absence of signs and symptoms of increased ICP

A

D. Absence of signs and symptoms of increased ICP

The primary goal after cranial surgery is prevention of increased ICP, and interventions to prevent ICP and infection postoperatively are nursing priorities. The residual deficits, rehabilitation potential, and ultimate function of the patient depend on the reason for surgery, the postoperative course, and the patient’s general state of health

144
Q

A patient is admitted to the hospital with possible bacterial meningitis. During the initial assessment, the nurse questions the patient about a recent history of

a. mosquito or tick bites
b. chickenpox or measles
c. cold sores or fever blisters
d. an upper respiratory infection

A

D. An upper respiratory infection

Meningitis is often a result of an upper respiratory infection or middle ear infection, where organisms gain entry to the CNS. Epidemic encephalitis is transmitted by ticks and mosquitoes, and nonepidemic encephalitis may occur as a complication of measles, chickenpox, or mumps. Encephalitis caused by the herpes simplex virus carries a high fatality rate

145
Q

Classic symptoms of bacterial meningitis include

a. papilledema and psychomotor seizures
b. high fever, nuchal rigidity, and severe headache
c. behavioral changes with memory loss and lethargy
d. positive Kernig’s and Brudzinski’s signs and hemiparesis

A

B. High fever, severe headache, nuchal rigidity
and positive Brudzinski’s and Kernig’s signs are such classic symptoms of meningitis that they are usually considered diagnostic for meningitis. Other symptoms, such as papilledema, generalized seizures, hemiparesis, and decreased LOC, may occur as complications of increased ICP and cranial nerve dysfunction.

146
Q

On physical examination of a patient with headache and fever, the nurse would suspect a brain abscess when the patient has

a. seizures
b. nuchal rigidity
c. focal symptoms
d. signs of increased ICP

A

c. focal symptoms

The symptoms of brain abscess closely resemble those of meningitis and encephalitis, including fever, headache, and increased ICP, except the patient also usually has some focal symptoms that reflect the local are of the abscess.

147
Q

Vasogenic cerebral edema increases ICP by

a. shifting fluid in gray matter
b. altering the endothelial lining of cerebral capillaries
c. leaking molecules from the intracellular fluid to the capillaries
d. altering the osmotic gradient flow inot the intravascular component

A

B. Altering the endothelial lining of cerebral capillaries

148
Q

A patient with ICP monitoring has pressure of 12 mm Hg. The nurse understand that this pressure reflects

a. a severe decrease in cerebral perfusion pressure
b. an alteration in the production of CSF
c. the loss of autoregulatory control of ICP
d. a normal balance between brain tissue, blood, and CSF

A

d. a normal balance between brain tissue, blood, and CSF

149
Q

The nurse plans care for a patient with increased ICP with the knowledge that the best way to position the patient is to

a. keep the head of the bed flat
b. elevate the head of the bed to 30 degrees
c. maintain patient on the left side with the head supported on a pillow
d. use a continuous rotation bed to continuously change patient position

A

b. elevate the head of the bed to 30 degrees

150
Q

The nurse is alerted to possible acute subdural hematoma in the patient who

a. has a linear skull fracture crossing a major artery
b. has focal symptoms of brain damage with no recollection of a head injury
c. develops decreased LOC and a headache within 48 hours of head injury
d. has an immediate loss of consciousness with a brief lucid interval followed by decreasing LOC

A

c. develops decreased LOC and a headache within 48 hours of head injury

151
Q

During admission of a patient with a severe head injury to the ED, the nurse places highest priority on assessment for

a. patency of of airway
b. presence of a neck injury
c. neurologic status with Glascow Coma Scale
d. CSF leakage from ears and nose

A

a. patency of of airway

#1 priority with all head injuries

152
Q

A patient is suspected of having a cranial tumor. The signs and symptoms include memory deficits, visual disturbances, weakness of right upper and lower extremities, and personality changes. The nurse recognizes that the tumor is most likely located in the

a. frontal lobe
b. parietal lobe
c. occipital lobe
d. temporal lobe

A

a. frontal lobe

153
Q

Nursing management of a patient with a brain tumor includes (select all that apply)

a. discussing with the patient methods to control appropriate behavior
b. using diversion techniques to leep the patient stimulated and motivated
c. assisting and supporting the family in understanding any changes in behavior
d. limiting self care activities until the patient has regained maximal physical functioning
e. plan for seizure precautions and teaching the patient and caregiver about antiseizure drugs.

A

c. assisting and supporting the family in understanding any changes in behavior
e. plan for seizure precautions and teaching the patient and caregiver about antiseizure drugs.

154
Q

The nurse on the clinical unit is assigned to four patients. Which patient should she assess first?

a. patient with a skull fracture whose nose is bleeding
b. elderly patient with a stroke who is confused and whose daughter is present
c. patient with meningitis who is suddenly agitated and reporting a HA of 10 on a 0 to 10 scale
d. patient who had a craniotomy for a brain tumor who is now 3 days postoperative and has had continued emesis

A

c. patient with meningitis who is suddenly agitated and reporting a HA of 10 on a 0 to 10 scale

155
Q

A nursing measure that is indicated to reduce the potential for seizures and increased ICP in the patient with bacterial meningitis is

a. administering codeine for relief of head and neck pain
b. controlling fever with prescribed drugs and cooling techniques
c. keeping the room darkened and quiet to minimize environmental stimulation
d. maintaining the patient on strict bed rest with the HOB slightly elevated

A

B. Controlling fever with prescribed drugs and cooling techniques

156
Q

After a patient experienced a brief episode of tinnitus, diplopia, and dysarthria with no residual effects, the nurse anticipates teaching the patient about

a. cerebral aneurysm clipping.
b. heparin intravenous infusion.
c. oral low-dose aspirin therapy.
d. tissue plasminogen activator (tPA).

A

c. oral low-dose aspirin therapy.

The patient’s symptoms are consistent with transient ischemic attack (TIA), and drugs that
inhibit platelet aggregation are prescribed after a TIA to prevent stroke. Continuous heparin
infusion is not routinely used after TIA or with acute ischemic stroke. The patient’s symptoms
are not consistent with a cerebral aneurysm. tPA is used only for acute ischemic stroke, not
for TIA.

157
Q

A 68-year-old patient is being admitted with a possible stroke. Which information from the assessment indicates that the nurse should consult with the health care provider before giving the prescribed aspirin?

a. The patient has dysphasia.
b. The patient has atrial fibrillation.
c. The patient reports that symptoms began with a severe headache.
d. The patient has a history of brief episodes of right-sided hemiplegia.

A

c. The patient reports that symptoms began with a severe headache.

A sudden onset headache is typical of a subarachnoid hemorrhage, and aspirin is
contraindicated. Atrial fibrillation, dysphasia, and transient ischemic attack (TIA) are not
contraindications to aspirin use, so the nurse can administer the aspirin

158
Q

During the change of shift report a nurse is told that a patient has an occluded left posterior cerebral artery. The nurse will anticipate that the patient may have

a. dysphasia.
b. confusion.
c. visual deficits.
d. poor judgment

A

c. visual deficits.

Visual disturbances are expected with posterior cerebral artery occlusion. Aphasia occurs with
middle cerebral artery involvement. Cognitive deficits and changes in judgment are more
typical of anterior cerebral artery occlusion

159
Q

When teaching about clopidogrel (Plavix), the nurse will tell the patient with cerebral atherosclerosis

a. to monitor and record the blood pressure daily.
b. that Plavix will dissolve clots in the cerebral arteries.
c. that Plavix will reduce cerebral artery plaque formation.
d. to call the health care provider if stools are bloody or tarry.

A

d. to call the health care provider if stools are bloody or tarry.

Clopidogrel (Plavix) inhibits platelet function and increases the risk for gastrointestinal bleeding, so patients should be advised to notify the health care provider about any signs of bleeding. The medication does not lower blood pressure, decrease plaque formation, or dissolve clots

160
Q

A patient with carotid atherosclerosis asks the nurse to describe a carotid endarterectomy. Which response by the nurse is accurate?

a. “The obstructing plaque is surgically removed from an artery in the neck.”
b. “The diseased portion of the artery in the brain is replaced with a synthetic graft.”
c. “A wire is threaded through an artery in the leg to the clots in the carotid artery and
the clots are removed.”
d. “A catheter with a deflated balloon is positioned at the narrow area, and the
balloon is inflated to flatten the plaque.

A

a. “The obstructing plaque is surgically removed from an artery in the neck.”

In a carotid endarterectomy, the carotid artery is incised and the plaque is removed. The
response beginning, “The diseased portion of the artery in the brain is replaced” describes an
arterial graft procedure. The answer beginning, “A catheter with a deflated balloon is
positioned at the narrow area” describes an angioplasty. The final response beginning, “A
wire is threaded through the artery” describes the mechanical embolus removal in cerebral
ischemia (MERCI) procedure.

161
Q

A patient admitted with possible stroke has been aphasic for 3 hours and his current blood pressure (BP) is 174/94 mm Hg. Which order by the health care provider should the nurse question?

a. Keep head of bed elevated at least 30 degrees.
b. Infuse normal saline intravenously at 75 mL/hr.
c. Administer tissue plasminogen activator (tPA) per protocol.
d. Administer a labetalol (Normodyne) drip to keep BP less than 140/90 mm Hg.

A

d. Administer a labetalol (Normodyne) drip to keep BP less than 140/90 mm Hg.

Because elevated BP may be a protective response to maintain cerebral perfusion,
antihypertensive therapy is recommended only if mean arterial pressure (MAP) is >130 mm
Hg or systolic pressure is >220 mm Hg. Fluid intake should be 1500 to 2000 mL daily to
maintain cerebral blood flow. The head of the bed should be elevated to at least 30 degrees,
unless the patient has symptoms of poor tissue perfusion. tPA may be administered if the
patient meets the other criteria for tPA use.

162
Q

A 56-year-old patient arrives in the emergency department with hemiparesis and dysarthria that started 2 hours previously, and health records show a history of several transient ischemic attacks (TIAs). The nurse anticipates preparing the patient for

a. surgical endarterectomy.
b. transluminal angioplasty.
c. intravenous heparin administration.
d. tissue plasminogen activator (tPA) infusion

A

d. tissue plasminogen activator (tPA) infusion

The patient’s history and clinical manifestations suggest an acute ischemic stroke and a
patient who is seen within 4.5 hours of stroke onset is likely to receive tPA (after screening
with a CT scan). Heparin administration in the emergency phase is not indicated. Emergent
carotid transluminal angioplasty or endarterectomy is not indicated for the patient who is
having an acute ischemic stroke

163
Q

For a patient who had a right hemisphere stroke the nurse establishes a nursing diagnosis of

a. risk for injury related to denial of deficits and impulsiveness.
b. impaired physical mobility related to right-sided hemiplegia.
c. impaired verbal communication related to speech-language deficits.
d. ineffective coping related to depression and distress about disability

A

a. risk for injury related to denial of deficits and impulsiveness.

The patient with right-sided brain damage typically denies any deficits and has poor impulse control, leading to risk for injury when the patient attempts activities such as transferring from a bed to a chair. Right-sided brain damage causes left hemiplegia.
Left-sided brain damage typically causes language deficits. Left-sided brain damage is associated with depression and
distress about the disability.

164
Q

When caring for a patient with a new right-sided homonymous hemianopsia resulting from a stroke, which intervention should the nurse include in the plan of care?

a. Apply an eye patch to the right eye.
b. Approach the patient from the right side.
c. Place objects needed on the patient’s left side.
d. Teach the patient that the left visual deficit will resolve.

A

c. Place objects needed on the patient’s left side.

During the acute period, the nurse should place objects on the patient’s unaffected side.
Because there is a visual defect in the right half of each eye, an eye patch is not appropriate.
The patient should be approached from the left side. The visual deficit may not resolve,
although the patient can learn to compensate for the defect.

165
Q

A left-handed patient with left-sided hemiplegia has difficulty feeding himself. Which intervention should be included in the plan of care?

a. Provide a wide variety of food choices.
b. Provide oral care before and after meals.
c. Assist the patient to eat with the right hand.
d. Teach the patient the “chin-tuck” technique.

A

c. Assist the patient to eat with the right hand.

Because the nursing diagnosis indicates that the patient’s imbalanced nutrition is related to the
left-sided hemiplegia, the appropriate interventions will focus on teaching the patient to use the right hand for self-feeding. The other interventions are appropriate for patients with other etiologies for the imbalanced nutrition.

166
Q

A 40-year-old patient has a ruptured cerebral aneurysm and subarachnoid hemorrhage. Which intervention will be included in the care plan?

a. Apply intermittent pneumatic compression stockings.
b. Assist to dangle on edge of bed and assess for dizziness.
c. Encourage patient to cough and deep breathe every 4 hours.
d. Insert an oropharyngeal airway to prevent airway obstruction.

A

a. Apply intermittent pneumatic compression stockings.

The patient with a subarachnoid hemorrhage usually has minimal activity to prevent cerebral
vasospasm or further bleeding and is at risk for venous thromboembolism (VTE). Activities
such as coughing and sitting up that might increase intracranial pressure (ICP) or decrease
cerebral blood flow are avoided. Because there is no indication that the patient is unconscious,
an oropharyngeal airway is inappropriate.

167
Q

A 47-year-old patient will attempt oral feedings for the first time since having a stroke. The nurse should assess the gag reflex and then

a. order a varied pureed diet.
b. assess the patient’s appetite.
c. assist the patient into a chair.
d. offer the patient a sip of juice

A

c. assist the patient into a chair.

The patient should be as upright as possible before attempting feeding to make swallowing
easier and decrease aspiration risk. To assess swallowing ability, the nurse should initially
offer water or ice to the patient. Pureed diets are not recommended because the texture is too
smooth. The patient may have a poor appetite, but the oral feeding should be attempted
regardless.

168
Q

A male patient who has right-sided weakness after a stroke is making progress in learning to use the left hand for feeding and other activities. The nurse observes that when the patient’s wife is visiting, she feeds and dresses him. Which nursing diagnosis is most appropriate for the patient?

a. Interrupted family processes related to effects of illness of a family member
b. Situational low self-esteem related to increasing dependence on spouse for care
c. Disabled family coping related to inadequate understanding by patient’s spouse
d. Impaired nutrition: less than body requirements related to hemiplegia and aphasia

A

c. Disabled family coping related to inadequate understanding by patient’s spouse

The information supports the diagnosis of disabled family coping because the wife does not understand the rehabilitation program. There are no data supporting low self-esteem, and the patient is attempting independence. The data do not support an interruption in family processes because this may be a typical pattern for the couple. There is no indication that the patient has impaired nutrition.

169
Q

Several weeks after a stroke, a 50-year-old male patient has impaired awareness of bladder fullness, resulting in urinary incontinence. Which nursing intervention will be best to include in the initial plan for an effective bladder training program?

a. Limit fluid intake to 1200 mL daily to reduce urine volume.
b. Assist the patient onto the bedside commode every 2 hours.
c. Perform intermittent catheterization after each voiding to check for residual urine.
d. Use an external “condom” catheter to protect the skin and prevent embarrassment.

A

b. Assist the patient onto the bedside commode every 2 hours.

Developing a regular voiding schedule will prevent incontinence and may increase patient
awareness of a full bladder. A 1200 mL fluid restriction may lead to dehydration. Intermittent
catheterization and use of a condom catheter are appropriate in the acute phase of stroke, but
should not be considered solutions for long-term management because of the risks for urinary
tract infection (UTI) and skin breakdown.
170
Q

A 72-year-old patient who has a history of a transient ischemic attack (TIA) has an order for aspirin 160 mg daily. When the nurse is administering medications, the patient says, “I don’t need the aspirin today. I don’t have a fever.” Which action should the nurse take?

a. Document that the aspirin was refused by the patient.
b. Tell the patient that the aspirin is used to prevent a fever.
c. Explain that the aspirin is ordered to decrease stroke risk.
d. Call the health care provider to clarify the medication order.

A

c. Explain that the aspirin is ordered to decrease stroke risk.

Aspirin is ordered to prevent stroke in patients who have experienced TIAs. Documentation of
the patient’s refusal to take the medication is an inadequate response by the nurse. There is no
need to clarify the order with the health care provider. The aspirin is not ordered to prevent
aches and pains.

171
Q

A patient in the clinic reports a recent episode of dysphasia and left-sided weakness at home that resolved after 2 hours. The nurse will anticipate teaching the patient about

a. alteplase (tPA).
b. aspirin (Ecotrin).
c. warfarin (Coumadin).
d. nimodipine (Nimotop)

A

b. aspirin (Ecotrin).

Following a transient ischemic attack (TIA), patients typically are started on medications such
as aspirin to inhibit platelet function and decrease stroke risk. tPA is used for acute ischemic
stroke. Coumadin is usually used for patients with atrial fibrillation. Nimodipine is used to
prevent cerebral vasospasm after a subarachnoid hemorrhage.

172
Q

A 58-year-old patient with a left-brain stroke suddenly bursts into tears when family members visit. The nurse should

a. use a calm voice to ask the patient to stop the crying behavior.
b. explain to the family that depression is normal following a stroke.
c. have the family members leave the patient alone for a few minutes.
d. teach the family that emotional outbursts are common after strokes.

A

d. teach the family that emotional outbursts are common after strokes.

Patients who have left-sided brain stroke are prone to emotional outbursts that are not
necessarily related to the emotional state of the patient. Depression after a stroke is common,
but the suddenness of the patient’s outburst suggests that depression is not the major cause of
the behavior. The family should stay with the patient. The crying is not within the patient’s
control and asking the patient to stop will lead to embarrassment.

173
Q

Which stroke risk factor for a 48-year-old male patient in the clinic is most important for the nurse to address?

a. The patient is 25 pounds above the ideal weight.
b. The patient drinks a glass of red wine with dinner daily.
c. The patient’s usual blood pressure (BP) is 170/94 mm Hg.
d. The patient works at a desk and relaxes by watching television.

A

c. The patient’s usual blood pressure (BP) is 170/94 mm Hg.

Hypertension is the single most important modifiable risk factor. People who drink more than
1 (for women) or 2 (for men) alcoholic beverages a day may increase risk for hypertension.
Physical inactivity and obesity contribute to stroke risk but not as much as hypertension.

174
Q

A patient in the emergency department with sudden-onset right-sided weakness is diagnosed with an intracerebral hemorrhage. Which information about the patient is most important to communicate to the health care provider?

a. The patient’s speech is difficult to understand.
b. The patient’s blood pressure is 144/90 mm Hg.
c. The patient takes a diuretic because of a history of hypertension.
d. The patient has atrial fibrillation and takes warfarin (Coumadin)

A

d. The patient has atrial fibrillation and takes warfarin (Coumadin)

The use of warfarin probably contributed to the intracerebral bleeding and remains a risk
factor for further bleeding. Administration of vitamin K is needed to reverse the effects of the
warfarin, especially if the patient is to have surgery to correct the bleeding. The history of
hypertension is a risk factor for the patient but has no immediate effect on the patient’s care.
The BP of 144/90 indicates the need for ongoing monitoring but not for any immediate
change in therapy. Slurred speech is consistent with a left-sided stroke, and no change in
therapy is indicated

175
Q

A patient with left-sided weakness that started 60 minutes earlier is admitted to the emergency department and diagnostic tests are ordered. Which test should be done first?

a. Complete blood count (CBC)
b. Chest radiograph (Chest x-ray)
c. 12-Lead electrocardiogram (ECG)
d. Noncontrast computed tomography (CT) scan

A

d. Noncontrast computed tomography (CT) scan

Rapid screening with a noncontrast CT scan is needed before administration of tissue
plasminogen activator (tPA), which must be given within 4.5 hours of the onset of clinical
manifestations of the stroke. The sooner the tPA is given, the less brain injury. The other
diagnostic tests give information about possible causes of the stroke and do not need to be
completed as urgently as the CT scan.

176
Q

Nurses in change-of-shift report are discussing the care of a patient with a stroke who has progressively increasing weakness and decreasing level of consciousness (LOC). Which nursing diagnosis do they determine has the highest priority for the patient?

a. Impaired physical mobility related to weakness
b. Disturbed sensory perception related to brain injury
c. Risk for impaired skin integrity related to immobility
d. Risk for aspiration related to inability to protect airway

A

d. Risk for aspiration related to inability to protect airway

Protection of the airway is the priority of nursing care for a patient having an acute stroke.
The other diagnoses are also appropriate, but interventions to prevent aspiration are the
priority at this time.

177
Q

Which information about the patient who has had a subarachnoid hemorrhage is most important to communicate to the health care provider?

a. The patient complains of having a stiff neck.
b. The patient’s blood pressure (BP) is 90/50 mm Hg.
c. The patient reports a severe and unrelenting headache.
d. The cerebrospinal fluid (CSF) report shows red blood cells (RBCs).

A

b. The patient’s blood pressure (BP) is 90/50 mm Hg.

To prevent cerebral vasospasm and maintain cerebral perfusion, blood pressure needs to be maintained at a level higher than 90 mm Hg systolic after a subarachnoid hemorrhage. A low BP or drop in BP indicates a need to administer fluids and/or vasopressors to increase the BP.
An ongoing headache, RBCs in the CSF, and a stiff neck are all typical clinical manifestations of a subarachnoid hemorrhage and do not need to be rapidly communicated to the health care
provider.

178
Q

The nurse is caring for a patient who has been experiencing stroke symptoms for 60 minutes. Which action can the nurse delegate to a licensed practical/vocational nurse (LPN/LVN)?

a. Assess the patient’s gag and cough reflexes.
b. Determine when the stroke symptoms began.
c. Administer the prescribed short-acting insulin.
d. Infuse the prescribed IV metoprolol (Lopressor).

A

c. Administer the prescribed short-acting insulin.

Administration of subcutaneous medications is included in LPN/LVN education and scope of
practice. The other actions require more education and scope of practice and should be done
by the registered nurse (RN).

179
Q

After receiving change-of-shift report on the following four patients, which patient should the
nurse see first?

a. A 60-year-old patient with right-sided weakness who has an infusion of tPA
prescribed
b. A 50-year-old patient who has atrial fibrillation and a new order for warfarin
(Coumadin)
c. A 40-year-old patient who experienced a transient ischemic attack yesterday who has a dose of aspirin due
d. A 30-year-old patient with a subarachnoid hemorrhage 2 days ago who has
nimodipine (Nimotop) scheduled

A

a. A 60-year-old patient with right-sided weakness who has an infusion of tPA
prescribed

tPA needs to be infused within the first few hours after stroke symptoms start in order to be
effective in minimizing brain injury. The other medications should also be given as quickly as
possible, but timing of the medications is not as critical.

180
Q

The nurse is caring for a patient who has just returned after having left carotid artery angioplasty and stenting. Which assessment information is of most concern to the nurse?

a. The pulse rate is 102 beats/min.
b. The patient has difficulty speaking.
c. The blood pressure is 144/86 mm Hg.
d. There are fine crackles at the lung bases

A

b. The patient has difficulty speaking.

Small emboli can occur during carotid artery angioplasty and stenting, and the aphasia indicates a possible stroke during the procedure. Slightly elevated pulse rate and blood pressure are not unusual because of anxiety associated with the procedure. Fine crackles at the lung bases may indicate atelectasis caused by immobility during the procedure. The nurse should have the patient take some deep breaths.

181
Q

A 70-year-old female patient with left-sided hemiparesis arrives by ambulance to the emergency department. Which action should the nurse take first?

a. Monitor the blood pressure.
b. Send the patient for a computed tomography (CT) scan.
c. Check the respiratory rate and effort.
d. Assess the Glasgow Coma Scale score.

A

c. Check the respiratory rate and effort.

The initial nursing action should be to assess the airway and take any needed actions to ensure
a patent airway. The other activities should take place quickly after the ABCs (airway,
breathing, and circulation) are completed.

182
Q

A 63-year-old patient who began experiencing right arm and leg weakness is admitted to the emergency department. In which order will the nurse implement these actions included in the stroke protocol?

a. Obtain computed tomography (CT) scan without contrast.
b. Infuse tissue plasminogen activator (tPA).
c. Administer oxygen to keep O2 saturation >95%.
d. Use National Institute of Health Stroke Scale to assess patient.

A

c. Administer oxygen to keep O2 saturation >95%.
d. Use National Institute of Health Stroke Scale to assess patient
a. Obtain computed tomography (CT) scan without contrast.
b. Infuse tissue plasminogen activator (tPA)

183
Q

The nurse determines that teaching about management of migraine headaches has been effective when the patient says which of the following?

A. “I can take the (Topamax) as soon as a headache starts.”
B. “A glass of wine might help me relax and prevent a headache.”
C. “I will lie down someplace dark and quiet when the headaches begin.”
D. “I should avoid taking aspirin and sumatriptan (Imitrex) at the same time.”

A

C. “I will lie down someplace dark and quiet when the headaches begin.”

It is recommended that the patient with a migraine rest in a dark, quiet area. Topiramate (Topamax) is used to prevent migraines and must be taken for several months to determine effectiveness. Aspirin or other nonsteroidal anti-inflammatory medications can be taken with the triptans. Alcohol may precipitate migraine headaches.

184
Q

The nurse will assess a 67-year-old patient who is experiencing a cluster headache for

a. nuchal rigidity.
b. unilateral ptosis.
c. projectile vomiting.
d. throbbing, bilateral facial pain.

A

b. unilateral ptosis.

Unilateral eye edema, tearing, and ptosis are characteristic of cluster headaches. Nuchal rigidity suggests meningeal irritation, such as occurs with meningitis. Although nausea and vomiting may occur with migraine headaches, projectile vomiting is more consistent with increased intracranial pressure (ICP). Unilateral sharp, stabbing pain, rather than throbbing pain, is characteristic of cluster headaches.

185
Q

While the nurse is transporting a patient on a stretcher to the radiology department, the patient begins having a tonic-clonic seizure. Which action should the nurse take?

a. Insert an oral airway during the seizure to maintain a patent airway.
b. Restrain the patient’s arms and legs to prevent injury during the seizure.
c. Time and observe and record the details of the seizure and postictal state.
d. Avoid touching the patient to prevent further nervous system stimulation.

A

c. Time and observe and record the details of the seizure and postictal state.

Because the diagnosis and treatment of seizures frequently are based on the description of the seizure, recording the length and details of the seizure is important. Insertion of an oral airway and restraining the patient during the seizure are contraindicated. The nurse may need to move the patient to decrease the risk of injury during the seizure.

186
Q

A high school teacher who has just been diagnosed with epilepsy after having a generalized tonic-clonic seizure tells the nurse, “I cannot teach anymore, it will be too upsetting if I have a seizure at work.” Which response by the nurse is best?

a. “You might benefit from some psychologic counseling.”
b. “Epilepsy usually can be well controlled with medications.”
c. “You will want to contact the Epilepsy Foundation for assistance.”
d. “The Department of Vocational Rehabilitation can help with work retraining.”

A

b. “Epilepsy usually can be well controlled with medications.”

The nurse should inform the patient that most patients with seizure disorders are controlled with medication. The other information may be necessary if the seizures persist after treatment with antiseizure medications is implemented.

187
Q

A patient has been taking phenytoin (Dilantin) for 2 years. Which action will the nurse take when evaluating for adverse effects of the medication?

a. Inspect the oral mucosa.
b. Listen to the lung sounds.
c. Auscultate the bowel tones.
d. Check pupil reaction to light.

A

a. Inspect the oral mucosa.

Phenytoin can cause gingival hyperplasia, but does not affect bowel tones, lung sounds, or pupil reaction to light.

188
Q

A patient reports feeling numbness and tingling of the left arm before experiencing a tonic-clonic seizure. The nurse determines that this history is consistent with what type of seizure?

a. Focal
b. Atonic
c. Absence
d. Myoclonic

A

a. Focal

The initial symptoms of a focal seizure involve clinical manifestations that are localized to a particular part of the body or brain. Symptoms of an absence seizure are staring and a brief loss of consciousness. In an atonic seizure, the patient loses muscle tone and (typically) falls to the ground. Myoclonic seizures are characterized by a sudden jerk of the body or extremities.

189
Q

When obtaining a health history and physical assessment for a 36-year-old female patient with possible multiple sclerosis (MS), the nurse should

a. assess for the presence of chest pain.
b. inquire about urinary tract problems.
c. inspect the skin for rashes or discoloration.
d. ask the patient about any increase in libido.

A

b. inquire about urinary tract problems.

Urinary tract problems with incontinence or retention are common symptoms of MS. Chest pain and skin rashes are not symptoms of MS. A decrease in libido is common with MS.

190
Q

A 31-year-old woman who has multiple sclerosis (MS) asks the nurse about risks associated with pregnancy. Which response by the nurse is accurate?

a. “MS symptoms may be worse after the pregnancy.”
b. “Women with MS frequently have premature labor.”
c. “MS is associated with an increased risk for congenital defects.”
d. “Symptoms of MS are likely to become worse during pregnancy.”

A

a. “MS symptoms may be worse after the pregnancy.”

During the postpartum period, women with MS are at greater risk for exacerbation of symptoms. There is no increased risk for congenital defects in infants born of mothers with MS. Symptoms of MS may improve during pregnancy. Onset of labor is not affected by MS.

191
Q

A 49-year-old patient with multiple sclerosis (MS) is to begin treatment with glatiramer acetate (Copaxone). Which information will the nurse include in patient teaching?

a. Recommendation to drink at least 4 L of fluid daily
b. Need to avoid driving or operating heavy machinery
c. How to draw up and administer injections of the medication
d. Use of contraceptive methods other than oral contraceptives

A

c. How to draw up and administer injections of the medication

Copaxone is administered by self-injection. Oral contraceptives are an appropriate choice for birth control. There is no need to avoid driving or drink large fluid volumes when taking glatiramer.

192
Q

Which information about a 60-year-old patient with MS indicates that the nurse should consult with the health care provider before giving the prescribed dose of dalfampridine (Ampyra)?

a. The patient has relapsing-remitting MS.
b. The patient walks a mile a day for exercise.
c. The patient complains of pain with neck flexion.
d. The patient has an increased serum creatinine level.

A

d. The patient has an increased serum creatinine level.

Dalfampridine should not be given to patients with impaired renal function. The other information will not impact whether the dalfampridine should be administered.

193
Q

Which action will the nurse plan to take for a 40-year-old patient with multiple sclerosis (MS) who has urinary retention caused by a flaccid bladder?

a. Decrease the patient’s evening fluid intake.
b. Teach the patient how to use the Credé method.
c. Suggest the use of adult incontinence briefs for nighttime only.
d. Assist the patient to the commode every 2 hours during the day.

A

b. Teach the patient how to use the Credé method.

The Credé method can be used to improve bladder emptying. Decreasing fluid intake will not improve bladder emptying and may increase risk for urinary tract infection (UTI) and dehydration. The use of incontinence briefs and frequent toileting will not improve bladder emptying.

194
Q

A 73-year-old patient with Parkinson’s disease has a nursing diagnosis of impaired physical mobility related to bradykinesia. Which action will the nurse include in the plan of care?

a. Instruct the patient in activities that can be done while lying or sitting.
b. Suggest that the patient rock from side to side to initiate leg movement.
c. Have the patient take small steps in a straight line directly in front of the feet.
d. Teach the patient to keep the feet in contact with the floor and slide them forward.

A

b. Suggest that the patient rock from side to side to initiate leg movement.

Rocking the body from side to side stimulates balance and improves mobility. The patient will be encouraged to continue exercising because this will maintain functional abilities. Maintaining a wide base of support will help with balance. The patient should lift the feet and avoid a shuffling gait.

195
Q

A 62-year-old patient who has Parkinson’s disease is taking bromocriptine (Parlodel). Which information obtained by the nurse may indicate a need for a decrease in the dose?

a. The patient has a chronic dry cough.
b. The patient has four loose stools in a day.
c. The patient develops a deep vein thrombosis.
d. The patient’s blood pressure is 92/52 mm Hg.

A

d. The patient’s blood pressure is 92/52 mm Hg.

Hypotension is an adverse effect of bromocriptine, and the nurse should check with the health care provider before giving the medication. Diarrhea, cough, and deep vein thrombosis are not associated with bromocriptine use.

196
Q

The nurse advises a patient with myasthenia gravis (MG) to

a. perform physically demanding activities early in the day.
b. anticipate the need for weekly plasmapheresis treatments.
c. do frequent weight-bearing exercise to prevent muscle atrophy.
d. protect the extremities from injury due to poor sensory perception.

A

a. perform physically demanding activities early in the day.

Muscles are generally strongest in the morning, and activities involving muscle activity should be scheduled then. Plasmapheresis is not routinely scheduled, but is used for myasthenia crisis or for situations in which corticosteroid therapy must be avoided. There is no decrease in sensation with MG, and muscle atrophy does not occur because although there is muscle weakness, they are still used.

197
Q

Which medication taken by a patient with restless legs syndrome should the nurse discuss with the patient?

a. Multivitamin (Stresstabs)
b. Acetaminophen (Tylenol)
c. Ibuprofen (Motrin, Advil)
d. Diphenhydramine (Benadryl)

A

d. Diphenhydramine (Benadryl)

Antihistamines can aggravate restless legs syndrome. The other medications will not contribute to restless legs syndrome.

198
Q

A 64-year-old patient who has amyotrophic lateral sclerosis (ALS) is hospitalized with pneumonia. Which nursing action will be included in the plan of care?

a. Assist with active range of motion (ROM).
b. Observe for agitation and paranoia.
c. Give muscle relaxants as needed to reduce spasms.
d. Use simple words and phrases to explain procedures.

A

a. Assist with active range of motion (ROM).

ALS causes progressive muscle weakness, but assisting the patient to perform active ROM will help maintain strength as long as possible. Psychotic manifestations such as agitation and paranoia are not associated with ALS. Cognitive function is not affected by ALS, and the patient’s ability to understand procedures will not be impaired. Muscle relaxants will further increase muscle weakness and depress respirations.

199
Q

A 40-year-old patient is diagnosed with early Huntington’s disease (HD). When teaching the patient, spouse, and children about this disorder, the nurse will provide information about the

a. use of levodopa-carbidopa (Sinemet) to help reduce HD symptoms.
b. prophylactic antibiotics to decrease the risk for aspiration pneumonia.
c. option of genetic testing for the patient’s children to determine their own HD risks.
d. lifestyle changes of improved nutrition and exercise that delay disease

A

c. option of genetic testing for the patient’s children to determine their own HD risks.

Genetic testing is available to determine whether an asymptomatic individual has the HD gene. The patient and family should be informed of the benefits and problems associated with genetic testing. Sinemet will increase symptoms of HD because HD involves an increase in dopamine. Antibiotic therapy will not reduce the risk for aspiration. There are no effective treatments or lifestyle changes that delay the progression of symptoms in HD.

200
Q

When a 74-year-old patient is seen in the health clinic with new development of a stooped posture, shuffling gait, and pill rolling-type tremor, the nurse will anticipate teaching the patient about

a. oral corticosteroids.
b. antiparkinsonian drugs.
c. magnetic resonance imaging (MRI).
d. electroencephalogram (EEG) testing.

A

b. antiparkinsonian drugs.

The diagnosis of Parkinson’s is made when two of the three characteristic manifestations of tremor, rigidity, and bradykinesia are present. The confirmation of the diagnosis is made on the basis of improvement when antiparkinsonian drugs are administered. This patient has symptoms of tremor and bradykinesia. The next anticipated step will be treatment with medications. MRI and EEG are not useful in diagnosing Parkinson’s disease, and corticosteroid therapy is not used to treat it.

201
Q

A 22-year-old patient seen at the health clinic with a severe migraine headache tells the nurse about having other similar headaches recently. Which initial action should the nurse take?

a. Teach about the use of triptan drugs.
b. Refer the patient for stress counseling.
c. Ask the patient to keep a headache diary.
d. Suggest the use of muscle-relaxation techniques.

A

c. Ask the patient to keep a headache diary.

The initial nursing action should be further assessment of the precipitating causes of the headaches, quality, and location of pain, etc. Stress reduction, muscle relaxation, and the triptan drugs may be helpful, but more assessment is needed first.

202
Q

A hospitalized patient complains of a bilateral headache, 4/10 on the pain scale, that radiates from the base of the skull. Which prescribed PRN medications should the nurse administer initially?

a. Lorazepam (Ativan)
b. Acetaminophen (Tylenol)
c. Morphine sulfate (Roxanol)
d. Butalbital and aspirin (Fiorinal)

A

b. Acetaminophen (Tylenol)

The patient’s symptoms are consistent with a tension headache, and initial therapy usually involves a nonopioid analgesic such as acetaminophen, which is sometimes combined with a sedative or muscle relaxant. Lorazepam may be used in conjunction with acetaminophen but would not be appropriate as the initial monotherapy. Morphine sulfate and butalbital and aspirin would be more appropriate for a headache that did not respond to a nonopioid analgesic.

203
Q

A 46-year-old patient tells the nurse about using acetaminophen (Tylenol) several times every day for recurrent bilateral headaches. Which action will the nurse plan to take first?

a. Discuss the need to stop taking the acetaminophen.
b. Suggest the use of biofeedback for headache control.
c. Describe the use of botulism toxin (Botox) for headaches.
d. Teach the patient about magnetic resonance imaging (MRI).

A

a. Discuss the need to stop taking the acetaminophen.

The headache description suggests that the patient is experiencing medication overuse headache. The initial action will be withdrawal of the medication. The other actions may be needed if the headaches persist.

204
Q

The health care provider is considering the use of sumatriptan (Imitrex) for a 54-year-old male patient with migraine headaches. Which information obtained by the nurse is most important to report to the health care provider?

a. The patient drinks 1 to 2 cups of coffee daily.
b. The patient had a recent acute myocardial infarction.
c. The patient has had migraine headaches for 30 years.
d. The patient has taken topiramate (Topamax) for 2 months.

A

b. The patient had a recent acute myocardial infarction.

The triptans cause coronary artery vasoconstriction and should be avoided in patients with coronary artery disease. The other information will be reported to the health care provider, but none of it indicates that sumatriptan would be an inappropriate treatment.

205
Q

The nurse observes a patient ambulating in the hospital hall when the patient’s arms and legs suddenly jerk and the patient falls to the floor. The nurse will first

a. assess the patient for a possible head injury.
b. give the scheduled dose of divalproex (Depakote).
c. document the timing and description of the seizure.
d. notify the patient’s health care provider about the seizure.

A

a. assess the patient for a possible head injury.

The patient who has had a myoclonic seizure and fall is at risk for head injury and should first be evaluated and treated for this possible complication. Documentation of the seizure, notification of the seizure, and administration of antiseizure medications are also appropriate actions, but the initial action should be assessment for injury.

206
Q

Which prescribed intervention will the nurse implement first for a patient in the emergency department who is experiencing continuous tonic-clonic seizures?

a. Give phenytoin (Dilantin) 100 mg IV.
b. Monitor level of consciousness (LOC).
c. Obtain computed tomography (CT) scan.
d. Administer lorazepam (Ativan) 4 mg IV.

A

d. Administer lorazepam (Ativan) 4 mg IV.

To prevent ongoing seizures, the nurse should administer rapidly acting antiseizure medications such as the benzodiazepines. A CT scan is appropriate, but prevention of any seizure activity during the CT scan is necessary. Phenytoin will also be administered, but it is not rapidly acting. Patients who are experiencing tonic-clonic seizures are nonresponsive, although the nurse should assess LOC after the seizure.

207
Q

The home health registered nurse (RN) is planning care for a patient with a seizure disorder related to a recent head injury. Which nursing action can be delegated to a licensed practical/vocational nurse (LPN/LVN)?

a. Make referrals to appropriate community agencies.
b. Place medications in the home medication organizer.
c. Teach the patient and family how to manage seizures.
d. Assess for use of medications that may precipitate seizures.

A

b. Place medications in the home medication organizer.

LPN/LVN education includes administration of medications. The other activities require RN education and scope of practice.

208
Q

A 76-year-old patient is being treated with carbidopa/levodopa (Sinemet) for Parkinson’s disease. Which information is most important for the nurse to report to the health care provider?

a. Shuffling gait
b. Tremor at rest
c. Cogwheel rigidity of limbs
d. Uncontrolled head movement

A

d. Uncontrolled head movement

Dyskinesia is an adverse effect of the Sinemet, indicating a need for a change in medication or decrease in dose. The other findings are typical with Parkinson’s disease.

209
Q

Which nursing diagnosis is of highest priority for a patient with Parkinson’s disease who is unable to move the facial muscles?

a. Activity intolerance
b. Self-care deficit: toileting
c. Ineffective self-health management
d. Imbalanced nutrition: less than body requirements

A

d. Imbalanced nutrition: less than body requirements

The data about the patient indicate that poor nutrition will be a concern because of decreased swallowing. The other diagnoses may also be appropriate for a patient with Parkinson’s disease, but the data do not indicate that they are current problems for this patient.

210
Q

Which assessment is most important for the nurse to make regarding a patient with myasthenia gravis?

a. Pupil size
b. Grip strength
c. Respiratory effort
d. Level of consciousness

A

c. Respiratory effort

Because respiratory insufficiency may be life threatening, it will be most important to monitor respiratory function. The other data also will be assessed but are not as critical.

211
Q

Following a thymectomy, a 62-year-old male patient with myasthenia gravis receives the usual dose of pyridostigmine (Mestinon). An hour later, the patient complains of nausea and severe abdominal cramps. Which action should the nurse take first?

a. Auscultate the patient’s bowel sounds.
b. Notify the patient’s health care provider.
c. Administer the prescribed PRN antiemetic drug.
d. Give the scheduled dose of prednisone (Deltasone).

A

b. Notify the patient’s health care provider.

The patient’s history and symptoms indicate a possible cholinergic crisis. The health care provider should be notified immediately, and it is likely that atropine will be prescribed. The other actions will be appropriate if the patient is not experiencing a cholinergic crisis.

212
Q

A hospitalized 31-year-old patient with a history of cluster headache awakens during the night with a severe stabbing headache. Which action should the nurse take first?

a. Start the ordered PRN oxygen at 6 L/min.
b. Put a moist hot pack on the patient’s neck.
c. Give the ordered PRN acetaminophen (Tylenol).
d. Notify the patient’s health care provider immediately.

A

a. Start the ordered PRN oxygen at 6 L/min.

Acute treatment for cluster headache is administration of 100% oxygen at 6 to 8 L/min. If the patient obtains relief with the oxygen, there is no immediate need to notify the health care provider. Cluster headaches last only 60 to 90 minutes, so oral pain medications have minimal effect. Hot packs are helpful for tension headaches but are not as likely to reduce pain associated with a cluster headache.

213
Q

Which intervention will the nurse include in the plan of care for a patient with primary restless legs syndrome (RLS) who is having difficulty sleeping?

a. Teach about the use of antihistamines to improve sleep.
b. Suggest that the patient exercise regularly during the day.
c. Make a referral to a massage therapist for deep massage of the legs.
d. Assure the patient that the problem is transient and likely to resolve.

A

b. Suggest that the patient exercise regularly during the day.

Nondrug interventions such as getting regular exercise are initially suggested to improve sleep quality in patients with RLS. Antihistamines may aggravate RLS. Massage does not alleviate RLS symptoms and RLS is likely to progress in most patients.

214
Q

Which information about a 72-year-old patient who has a new prescription for phenytoin (Dilantin) indicates that the nurse should consult with the health care provider before administration of the medication?

a. Patient has generalized tonic-clonic seizures.
b. Patient experiences an aura before seizures.
c. Patient’s most recent blood pressure is 156/92 mm Hg.
d. Patient has minor elevations in the liver function tests.

A

d. Patient has minor elevations in the liver function tests.

Many older patients (especially with compromised liver function) may not be able to metabolize phenytoin. The health care provider may need to choose another antiseizure medication. Phenytoin is an appropriate medication for patients with tonic-clonic seizures, with or without an aura. Hypertension is not a contraindication for phenytoin therapy.

215
Q

After change-of-shift report, which patient should the nurse assess first?

a. Patient with myasthenia gravis who is reporting increased muscle weakness
b. Patient with a bilateral headache described as “like a band around my head”
c. Patient with seizures who is scheduled to receive a dose of phenytoin (Dilantin)
d. Patient with Parkinson’s disease who has developed cogwheel rigidity of the arms

A

a. Patient with myasthenia gravis who is reporting increased muscle weakness

Because increased muscle weakness may indicate the onset of a myasthenic crisis, the nurse should assess this patient first. The other patients should also be assessed, but do not appear to need immediate nursing assessments or actions to prevent life-threatening complications.

216
Q

A 27-year-old patient who has been treated for status epilepticus in the emergency department will be transferred to the medical nursing unit. Which equipment should the nurse have available in the patient’s assigned room (select all that apply)?

a. Side-rail pads
b. Tongue blade
c. Oxygen mask
d. Suction tubing
e. Urinary catheter
f. Nasogastric tube

A

a. Side-rail pads
c. Oxygen mask
d. Suction tubing

The patient is at risk for further seizures, and oxygen and suctioning may be needed after any seizures to clear the airway and maximize oxygenation. The bed’s side rails should be padded to minimize the risk for patient injury during a seizure. Use of tongue blades during a seizure is contraindicated. Insertion of a nasogastric (NG) tube is not indicated because the airway problem is not caused by vomiting or abdominal distention. A urinary catheter is not required unless there is urinary retention

217
Q

A patient with Parkinson’s disease is admitted to the hospital for treatment of pneumonia. Which nursing interventions will be included in the plan of care (select all that apply)?

a. Use an elevated toilet seat.
b. Cut patient’s food into small pieces.
c. Provide high-protein foods at each meal.
d. Place an armchair at the patient’s bedside.
e. Observe for sudden exacerbation of symptoms.

A

a. Use an elevated toilet seat.
b. Cut patient’s food into small pieces.
d. Place an armchair at the patient’s bedside.

Because the patient with Parkinson’s has difficulty chewing, food should be cut into small pieces. An armchair should be used when the patient is seated so that the patient can use the arms to assist with getting up from the chair. An elevated toilet seat will facilitate getting on and off the toilet. High-protein foods will decrease the effectiveness of L-dopa. Parkinson’s is a steadily progressive disease without acute exacerbations.